PRNU 129 PrepU questions

Ace your homework & exams now with Quizwiz!

A pregnant woman asks the nurse, "I'm a big coffee drinker. Will the caffeine in my coffee hurt my baby?" Which response by the nurse would be most appropriate?

"If you keep your intake to less than 200 mg/day, you should be okay." Explanation: The effect of caffeine intake during pregnancy on fetal growth and development is still unclear. A recent study found that caffeine intake of no more than 200 mg/day during pregnancy does not affect pregnancy duration and the condition of the newborn. Birth defects have not been linked to caffeine consumption, but maternal coffee consumption decreases iron absorption and may increase the risk of anemia during pregnancy. It is not known if there is a correlation between high caffeine intake and miscarriage due to lack of sufficient studies.

As a woman enters the second stage of labor, her membranes spontaneously rupture. When this occurs, what would the nurse do next?

Assess fetal heart rate for fetal safety. Explanation: Rupture of the membranes may lead to a prolapsed cord. Assessment of FHR detects this.

Preterm premature rupture of membranes (PROM) can be a serious complication of labor. What is the most common cause of preterm PROM?

Infection

A woman with gestational hypertension develops eclampsia and experiences a seizure. Which intervention would the nurse identify as the priority?

oxygenation Explanation: As with any seizure, the priority is to clear the airway and maintain adequate oxygenation both to the mother and the fetus. Fluids and control of hypertension are addressed once the airway and oxygenation are maintained. Delivery of fetus is determined once the seizures are controlled and the woman is stable.

A woman comes in for her annual gynecological examination and informs the nurse that she is going to try and become pregnant. She asks the nurse when the best time in the month is to become pregnant. What is the best response by the nurse?

"3 days before until 2 days after ovulation." Sperm are able to fertilize the ovum for up to 72 hours after ejaculation, and the ovum remains fertile for a maximum of 48 hours after ovulation. Thus, the window of opportunity for conception is 3 days before until 2 days after ovulation.

A nurse is providing care to a postpartum woman who gave birth about 2 days ago. The client asks the nurse, "I haven't moved my bowels yet. Is this a problem?" Which response by the nurse would be most appropriate?

"It might take up to a week for your bowels return to their normal pattern." Spontaneous bowel movements may not occur for 1 to 3 days after giving birth because of a decrease in muscle tone in the intestines as a result of elevated progesterone levels. Normal patterns of bowel elimination usually return within a week after birth. The nurse should assess the client's abdomen for bowel sounds and ascertain if the woman is passing gas to gain additional information. Obtaining an order for a laxative may be appropriate, but this response does not address the client's concern. Telling the client that it is unusual is inaccurate and could cause the client additional anxiety. Notifying the healthcare provider is not necessary, and this statement could add to the client's current concern.

A female client is concerned about developing breast cancer. To help alleviate this client's concern, the nurse assesses the client for risk factors. Which cancers would the nurse assess for in the client's family history?

Breast and peritoneal Explanation: The risk factors for breast cancer include a personal or family history of breast, ovarian, or peritoneal cancer. Other risk factors include a genetic predisposition, radiation to the chest from ages 10 to 30 years, first birth after 30 years, nulliparity, dense breasts, history of benign biopsy, and the use of exogenous estrogen.

A woman in labor is having very intense contractions with a resting uterine tone >20 mm Hg. The woman is screaming out every time she has a contraction. What is the highest priority fetal assessment the health care provider should focus on at this time?

Look for late decelerations on monitor, which is associated with fetal anoxia. Explanation: A danger of hypertonic contractions is that the lack of relaxation between contractions may not allow optimal uterine artery filling; this can lead to fetal anoxia early in the latent phase of labor. Applying a uterine and a fetal external monitor will help identify that the resting phase between contractions is adequate and that the FHR is not showing late deceleration.

Which respiratory disorder in a neonate is usually mild and runs a self-limited course?

Transient tachypnea Explanation: Transient tachypnea has an invariably favorable outcome after several hours to several days. The outcome of pneumonia depends on the causative agent involved and may have complications. Meconium aspiration, depending on severity, may have long-term adverse effects. In persistent pulmonary hypertension, mortality is more than 50%.

The nurse examines a 26-week-old premature neonate. The skin temperature is lowered. What could be a consequence of the infant being cold?

apnea Explanation: A premature neonate has thin skin, an immature central nervous system (CNS), lack of brown fat stores, and an increased weight-to-surface area ratio. These are predisposing thermoregulation problems that can lead to hypothermia. As a result, the infant may become apneic, have respiratory distress, increase his or her oxygen need, or be cyanotic. The other choices are not specific to increased oxygen demand or respiratory distress.

The nurse would monitor clients with which conditions for fetal demise in late pregnancy

prolonged pregnancy hypertension Explanation: Prolonged pregnancy and hypertension are causes of intrauterine fetal demise in late pregnancy that the nurse should be aware of. Other factors resulting in intrauterine fetal demise include infection, advanced maternal age, Rh disease, uterine rupture, diabetes, congenital anomalies, cord accident, abruption, premature rupture of membranes, or hemorrhage. Hydramnios, multifetal gestation, and malpresentation are not the causes of intrauterine fetal demise in late pregnancy; they are causes of umbilical cord prolapse.

Which would be a normal finding by the nurse during a physical exam of a woman in her third trimester?

Dyspnea Explanation: In the third trimester, a women experiences dyspnea from the uterus pushing up into the diaphragm. A pregnant woman will experience lordosis, not kyphosis. Ptyalism is excessive saliva production and is often seen in the first trimester of pregnancy. The hematocrit of a pregnant woman will decrease in the third trimester, not increase.

A preterm infant is placed on ventilatory assistance for respiratory distress syndrome. In light of her lung pathology, which additional ventilatory measure would you anticipate planning?

Positive end-expiratory pressure to increase oxygenation Explanation: Positive end-expiratory pressure, like expiratory grunting, prevents alveoli from fully closing on expiration and reduces the respiratory effort needed for inspiration.

A client is reporting considerable postpartum abdominal and perineal pain at a 7 on a scale of 1 to 10. The nurse will prioritize which action after noting the client is currently receiving ibuprofen 600 mg every 8 hours?

Ibuprofen (600 to 800 mg) may be ordered to be given every 6 to 8 hours around the clock. This type of dosing is often more effective at keeping pain under control than is an "as-needed" schedule; however, a combination medication, such as acetaminophen with codeine, may be ordered on an as-needed basis for breakthrough pain. Applying either a hot pad or cold pack or changing positions are nonpharmacologic approaches which may be used to help with the pain if the client does not desire to take medication. It may also lessen the amount of pain medication that is needed to control the client's pain.

A pregnant woman is diagnosed with iron-deficiency anemia and is prescribed an iron supplement. After teaching her about the prescribed iron supplement, which statement indicates successful teaching?

"I need to drink plenty of fluids to prevent constipation." Explanation: Iron supplements can lead to constipation, so the woman needs to increase her intake of fluids. Milk inhibits absorption and should be discouraged. Vitamin C-containing fluids such as orange juice are encouraged because they promote absorption. Ideally the woman should take the iron on an empty stomach to improve absorption, but many women cannot tolerate the gastrointestinal discomfort it causes. In such cases, the woman should take it with meals. Iron typically causes the stool to become black and tarry; there is no need for the woman to notify the health care provider.

The postpartum client and her husband are excited about their new baby. However, they are also concerned about getting pregnant again too soon and ask about using birth control. Which instruction should the nurse include in their discharge education to address this issue?

"Ovulation may return as soon as 3 weeks after birth." Ovulation may start at soon as 3 weeks after birth. The client needs to be aware and use a form of birth control. She needs to be cleared by her health care provider prior to intercourse if she has a vaginal birth, but in the event that she has intercourse, needs to be prepared for the possibility of pregnancy. Ovulation can occur without the return of the menstrual cycle, and ovulation does return sooner than 6 months after birth.

After teaching a class about intimate partner violence to a group of young adults, the nurse determines that additional teaching is needed based on which statement by the group?

"People who are battered typically come from lower socioeconomic groups." Explanation: Violence occurs in all socioeconomic classes, not just lower socioeconomic classes. Violence is a learned behavior and can be changed. Every state considers violence against women and children to be a crime. Intimate partner violence is a pattern of coercion and control that one person exerts over another and is repeated using a number of tactics.

A pregnant women calls the clinic to report a small amount of painless vaginal bleeding. What response by the nurse is best?

"Please come in now for an evaluation by your health care provider." Explanation: Bleeding during pregnancy is always a deviation from normal and should be evaluated carefully. It may be life-threatening or it may be something that is not a threat to the mother and/or fetus. Regardless, it needs to be evaluated quickly and carefully. Telling the client it may be harmless is a reassuring statement, but does not suggest the need for urgent evaluation. Having the mother lay on her left side and drink water is indicated for cramping.

A client at 32 weeks' gestation has recently been diagnosed with acute herpes type 2. The client asks what can happen to the baby as a result of this infection. How should the nurse bestrespond?

"There is a chance your baby may have a form of intellectual disability." Explanation: Herpes type 2 (genital herpes) contamination can occur during birth. The infant is susceptible to intellectual disability, premature birth, low birth weight, blindness, death as a result of this infection. The infant is not protected from the infection as it is contaminated during vaginal birth. Premature birth is a risk, not postdate birth.

A pregnant woman who has been taking penicillin prophylactically because she had rheumatic fever as a child tells the nurse that she wants to stop taking it now that she is pregnant. Which of the following is the best response by the nurse?

"You should continue taking this drug, because penicillin is not known to be a fetal teratogen." Explanation: A woman taking penicillin prophylactically because she had rheumatic fever as a child and wants to prevent a recurrence should continue this drug during pregnancy. Penicillin is not known to be a teratogen.

A woman who gave birth 24 hours ago tells the nurse, "I've been urinating so much over the past several hours." Which response by the nurse would be most appropriate?

"Your body is undergoing many changes that cause your bladder to fill quickly." Postpartum diuresis occurs as a result of several mechanisms: the large amounts of IV fluids given during labor, a decreasing antidiuretic effect of oxytocin as its level declines, the buildup and retention of extra fluids during pregnancy, and a decreasing production of aldosterone—the hormone that decreases sodium retention and increases urine production. All these factors contribute to rapid filling of the bladder within 12 hours of birth. Diuresis begins within 12 hours after childbirth and continues throughout the first week postpartum.

The nurse assesses the client and tells her the baby is at +1 station. Which is the best response by the nurse when asked by the client what this means concerning the location of the baby?

1 cm below the ischial spine. Explanation: A negative station is above the ischial spines, 0 station is at the ischial spines, and positive station is below the ischial spines. The symphysis pubis is used to determine fundal height during the pregnancy. It is also a landmark which can be used for determining urinary bladder status.

The nurse would be alert for the development of jaundice in a newborn at which age?

3-5 days Explanation: Jaundice is the yellowing of the skin caused by the deposit of bilirubin into subcutaneous tissue. This usually peaks in a newborn between the 3rd and 5th day of life.

Utilize the GTPAL system to classify a woman who is currently 18 weeks pregnant. This is her 4th pregnancy. She gave birth to one baby vaginally at 26 weeks who died, experienced a miscarriage, and has one living child who was delivered at 38 weeks gestation.

4, 1, 1, 1, 1 The GTPAL system is used to classifying pregnancy status. G = gravida, T= term, P = preterm, A = number of abortions, L= number of living children.

Hypoglycemia in a mature infant is defined as a blood glucose level below which amount?

40 mg/100 ml whole blood Explanation: Because newborns do not manifest symptoms of a reduced glucose level until it decreases well below adult levels, a finding below 40 mg/100 ml whole blood is considered hypoglycemia.

The nurse is preparing a mother for a planned cesarean birth. The nurse ascertains that the mother has previously had a deep vein thrombosis. Heparin is ordered prophylactically. The nurse determines this medication will be administered:

8 hours after birth. Explanation: All women undergoing a cesarean birth need mechanical thromboembolism prophylaxis during and after surgery until regular ambulation is established. Women with a history of thromboembolism have a higher risk for a blood clot and therefore require pharmacologic prophylaxis, usually with heparin. The nurse should start administering heparin 6 to 12 hours after the birth to minimize the chance for hemorrhage; heparin should continue until ambulation is well established. Giving heparin 1 hour after delivery would increase the chance for hemorrhage. Giving heparin 14 to 24 hours would be too late since the mother should be ambulating by that time.

A client gave birth to a child 3 hours ago and noticed a triangular-shaped gap in the bones at the back of the head of her newborn. The attending nurse informs the client that it is the posterior fontanel (fontanelle). The client is anxious to know when the posterior fontanel will close. Which time span is the normal duration for the closure of the posterior fontanel?

8 to 12 weeks Explanation: The posterior fontanel is a triangular-shaped area at the back of the skull. The nurse should inform the client that the posterior fontanelle normally closes by 8 to 12 weeks after birth, and if there is delay the primary health care provider should be notified.

A pregnant woman in her 39th week of pregnancy presents to the clinic with a vaginal infection. She tests positive for chlamydia. What would this disease make her infant at risk for?

A pregnant woman who contracts chlamydia is at increased risk for spontaneous abortion (miscarriage), preterm rupture of membranes, and preterm labor. The postpartum woman is at higher risk for endometritis (Fletcher & Ball, 2006). The fetus can encounter bacteria in the vagina during the birth process. If this happens, the newborn can develop pneumonia or conjunctivitis that can lead to blindness.

A pregnant client at 32 weeks' gestation is treated with magnesium sulfate for seizure management. The nurse assesses which of the following for evidence of magnesium toxicity?

Absence of knee jerk response Explanation: Magnesium sulfate toxicity is characterized by absence of deep tendon reflexes like the knee jerk reflex. Urinary retention, and not frequency of micturition, is seen with magnesium sulfate toxicity. Magnesium sulfate is given to treat seizures associated with hypertension and proteinuria in pregnancy, and therefore decreases the blood pressure. It does not cause an increase in blood pressure. There is respiratory depression, and not an increased rate of respiration, with magnesium sulfate toxicity.

At an amniocentesis just prior to birth, a fetus's lecithin/sphingomyelin ratio was determined to be 1:1. Based on this, she is prone to which type of respiratory problem following birth?

Alveolar collapse on expiration Explanation: Without adequate surfactant, infants are unable to sustain respiratory function and, thus, develop respiratory distress syndrome with alveolar collapse on expiration.

A pregnant client is undergoing a fetal biophysical profile. Which parameter of the profile helps measure long-term adequacy of the placental function?

Amniotic fluid volume Explanation: A biophysical profile combines five parameters (fetal reactivity, fetal breathing movements, fetal body movement, fetal tone, and amniotic fluid volume) into one assessment. The fetal heart and breathing record measures short-term central nervous system function; the amniotic fluid volume helps measure long-term adequacy of placental function.

A nurse is assessing a postpartal client. Which assessment finding would the nurse correlate to an increased loss of blood during and after delivery?

Anemia Pale conjunctiva Orthostatic hypotension Explanation: Blood loss greater than 250 to 500 mL after a vaginal delivery can produce findings such as anemia, pale conjunctiva, and orthostatic hypotension, especially if the woman's hemoglobin and hematocrit were not adequate before delivery. Elevated white count and excessive hair loss are unrelated to blood loss.

A nurse is caring for a postpartum client diagnosed with von Willebrand disease. What should be the nurse's priority for this client?

Check the lochia. Explanation: The nurse should assess the client for prolonged bleeding time. von Willebrand disease is a congenital bleeding disorder, inherited as an autosomal dominant trait, that is characterized by a prolonged bleeding time, a deficiency of von Willebrand factor, and impairment of platelet adhesion. A fever of 100.4° F (30.0° C) after the first 24 hours following birth and pain indicate infection. A client with a postpartum fundal height that is higher than expected may have subinvolution of the uterus.

A preterm newborn has developed necrotizing enterocolitis (NEC). Which of the following gastrointestinal (GI) characteristics would the nurse explain as predisposing a preterm newborn to NEC?

Diminished gastric enzymes Explanation: Diminished gastric enzymes in the preterm newborn predispose the newborn to NEC. In NEC, there is acute inflammation of the bowel associated with ischemia. This can lead to bowel necrosis and perforation. Preterm infants are at higher risk of developing NEC, due to gastric immaturity and increased risk of infections. The other GI characteristics predisposing preterm infants to NEC include decreased gut motility, delayed gastric emptying and weak esophageal sphincter tone.

The nurse is monitoring a woman who is receiving IV oxytocin to assist with uterine irritability. Which action should the nurse prioritize if the woman's contractions are determined to be 80 seconds in length after 1 hour of administration of the oxytocin?

Discontinue the oxytocin infusion. Explanation: If uterine contractions lengthen beyond 70 seconds, there is apt to be an interference with fetal circulation. Discontinuing the infusion allows contractions to shorten in length and allow fetal nourishment. The nurse would not increase the flow rate of the main line infusion or slow the infusion without the health care provider's prescription. Uterine contractions are monitored continuously.

While changing a female newborn's diaper, the nurse observes a mucus-like, slightly bloody vaginal discharge. Which action would the nurse do next?

Document this as pseudo menstruation. Explanation: The nurse should assess pseudomenstruation, a vaginal discharge composed of mucus mixed with blood, which may be present during the first few weeks of life. This discharge requires no treatment. The discharge is a normal finding and thus does not need to be reported immediately. It is not an indication of infection. The female genitalia normally will be engorged, so assessing for engorgement is not indicated.

A nurse is caring for an infant born with polycythemia. Which intervention is most appropriate when caring for this infant?

Focus on decreasing blood viscosity by increasing fluid volume. Explanation: The nurse should focus on decreasing blood viscosity by increasing fluid volume in the newborn with polycythemia. Checking blood glucose within 2 hours of birth by a reagent test strip and screening every 2 to 3 hours or before feeds are not interventions that will alleviate the condition of an infant with polycythemia. The nurse should monitor and maintain blood glucose levels when caring for a newborn with hypoglycemia, not polycythemia.

A pregnant client with sickle cell anemia is admitted in crisis. Which nursing intervention should the nurse prioritize?

IV fluids Explanation: A sickle cell crisis during pregnancy is usually managed by exchange transfusion, oxygen, and IV fluids. Antihypertensive drugs usually aren't necessary. Diuretics would not be used unless fluid overload resulted. The client would be given antibiotics only if there were evidence of an infection.

A pregnant client has been diagnosed with gonorrhea. Which nursing interventions should be performed to prevent gonococcal ophthalmia neonatorum in the baby?

Instill a prophylactic agent in the eyes of the newborn. Explanation: To prevent gonococcal ophthalmia neonatorum in the baby, the nurse should instill a prophylactic agent in the eyes of the newborn. Cephalosporins are administered to the mother during pregnancy to treat gonorrhea but not to prevent infection in the newborn. Performing a cesarean birth will not prevent gonococcal ophthalmia neonatorum in the newborn. An antiretroviral syrup is administered to the newborn only if the mother is human immunodeficiency virus-positive and will not help prevent gonococcal ophthalmia neonatorum in the baby.

How can pelvic inflammatory disease (PID) affect fertility?

It interferes with the transport of ova because of tubal scarring. Explanation: Pelvic inflammatory disease, or infection of the fallopian tubes, results in scarring and adhesions of the tubes, leading to poor transport of ova.

A woman who is about to be discharged after a vaginal birth notices a flea-like rash on her newborn's chest. The rash has tiny red lesions all across the nipple line. What does this rash indicate?

It is a normal skin finding in a newborn. Explanation: This rash is most likely is erythema toxicum, also known as newborn rash.

A client in her eighth month of pregnancy who has cardiac disease is experiencing profound shortness of breath and a cough that produces blood-speckled sputum, in addition to systemic hypotension. The nurse recognizes that this patient most likely is experiencing which condition?

Left-sided heart failure Explanation: In left-sided heart failure, the left ventricle cannot move the large volume of blood forward that it has received by the left atrium from the pulmonary circulation. It is characterized by a decrease in systemic blood pressure and pulmonary edema that produces profound shortness of breath. If pulmonary capillaries rupture under the pressure, small amounts of blood leak into the alveoli and the woman develops a productive cough with blood-speckled sputum. Right-sided heart failure is characterized by extreme liver enlargement, distention of abdominal and lower extremity vessels, ascites, and peripheral edema. A woman with peripartal cardiomyopathy develops signs of myocardial failure such as shortness of breath, chest pain, and nondependent edema. Her heart increases in size (cardiomegaly).The signs of a pulmonary embolism include chest pain, a sudden onset of dyspnea, a cough with hemoptysis, tachycardia or missed beats, or dizziness and fainting.

A postpartum client is showing signs and symptoms of a pulmonary embolism. What should the nurse do?

Raise the head of the bed to at least 45 degrees. Explanation: Immediate action is crucial for the woman who develops a pulmonary embolism. Immediately raise the head of the bed to at least 45 degrees to facilitate breathing. Begin oxygen therapy at 8 to 10 liters per minute via facemask and notify the health care provider.

Which intervention would be helpful to a client who is bottle feeding her infant and experiencing hard, engorged breasts?

applying ice Women who do not breastfeed often experience moderate to severe engorgement and breast pain when no treatment is applied. Ice promotes comfort by decreasing blood flow (vasoconstriction), numbing the area, and discouraging further letdown of milk. Restricting fluids does not reduce engorgement and should not be encouraged. Warm compresses will promote blood flow and hence, milk production, worsening the problem of engorgement. Bromocriptine has been removed from the market for lactation suppression.

A nurse is caring for a client in the clinic. Which sign or symptom may indicate that the client has gonorrhea?

burning on urination Explanation: Burning on urination may be a symptom of gonorrhea or urinary tract infection. A dry, hacking cough is a sign of a respiratory infection, not gonorrhea. A diffuse rash may indicate secondary stage syphilis. A painless chancre is the hallmark of primary syphilis. It appears wherever the organisms enter the body, such as on the genitalia, anus, or lips.

A woman is admitted to the labor suite with contractions every 5 minutes lasting 1 minute. She is postterm and has oligohydramnios. What does this increase the risk of during birth?

cord compression Explanation: Oligohydramnios and meconium staining of the amniotic fluid are common complications of postterm pregnancy. Oligohydramnios increases the incidence of cord compression, which can lead to fetal distress during labor.

Because of a family history of hypertension, a client chooses to not add any salt to the diet. The nurse instructs the client to consume what foods?

cranberries and seafood Explanation: Iodine is needed for thyroid function. Cranberries and seafood contain iodine. Yogurt, almonds, vegetables, eggs, organ meat, and dried fruit do not have adequate iodine to support thyroid function.

A woman received morphine during labor to help with pain control. Which finding would the nurse need to monitor the newborn for after birth?

decreased alertness Explanation: Morphine is a commonly used opioid for the management of pain during labor. It is associated with newborn respiratory depression, decreased alertness, inhibited sucking, and a delay in effective feeding.

A 29-year-old client has gestational diabetes. The nurse is teaching her about managing her glucose levels. Which therapy would be most appropriate for this client?

diet Explanation: Clients with gestational diabetes are usually managed by diet alone to control their glucose intolerance. Long-acting insulin usually is not needed for blood glucose control in the client with gestational diabetes. Oral hypoglycemic drugs are contraindicated in pregnancy. Glucagon raises blood glucose and is used to treat hypoglycemic reactions.

The nurse explains to a pregnant client that she will need to take iron during her pregnancy after being diagnosed with iron-deficiency anemia. The nurse suggests that absorption of the supplemental iron can be increased by taking it with which substance?

orange juice Explanation: Anemia is a condition in which the blood is deficient in red blood cells, from an underlying cause. The woman needs to take iron to manufacture enough red blood cells. Taking an iron supplement will help improve her iron levels, and taking iron with foods containing ascorbic acid, such as orange juice, improves the absorption of iron.

As part of a 31-year-old client's prenatal care, the nurse is assessing immunization history. Which immunization is most relevant to ensuring a healthy fetus?

rubella Explanation: Maternal exposure to rubella during pregnancy poses a particular fetal risk that supersedes the significance of hepatitis, measles, diphtheria, tetanus, or pertussis.

The nurse is providing care to several pregnant women who may be scheduled for labor induction. The nurse identifies the woman with which Bishop score as having the best chance for a successful induction and vaginal birth?

11 Explanation: The Bishop score helps identify women who would be most likely to achieve a successful induction. The duration of labor is inversely correlated with the Bishop score: a score over 8 indicates a successful vaginal birth. Therefore the woman with a Bishop score of 11 would have the greatest chance for success. Bishop scores of less than 6 usually indicate that a cervical ripening method should be used prior to induction.

A postpartum woman is developing a thrombophlebitis in her right leg. Which assessments would the nurse make to detect this?

Assess for pedal edema. Explanation: Calf swelling, erythema, warmth, tenderness, and pedal edema may be noted and are caused by an inflammatory process and obstruction of venous return.

A couple wants to start a family. They are concerned that their child will be at risk for cystic fibrosis because they each have a cousin with cystic fibrosis. They are seeing a nurse practitioner for preconceptual counseling. What would the nurse practitioner tell them about cystic fibrosis?

It is an autosomal recessive disorder. Cystic fibrosis is autosomal recessive. Nurses also consider other issues when assessing the risk for genetic conditions in couples and families. For example, when obtaining a preconception or prenatal family history, the nurse asks if the prospective parents have common ancestors. This is important to know because people who are related have more genes in common than those who are unrelated, thus increasing their chance for having children with autosomal recessive inherited condition such as cystic fibrosis. Mitochondrial inheritance occurs with defects in energy conversion and affects the nervous system, kidney, muscle, and liver. X-linked inheritance, which has been inherited from a mutant allele of the mother, affects males. Autosomal dominant is an X-linked dominant genetic disease.

Which intervention is helpful for the neonate experiencing drug withdrawal?

Place the Isolette in a quiet area of the nursery. Explanation: Neonates experiencing drug withdrawal commonly have sleep disturbance. The neonate should be moved to a quiet area of the nursery to minimize environmental stimuli. Medications, such as phenobarbital and paregoric, should be given as needed. The neonate should be swaddled to prevent him from flailing and stimulating himself.

A postpartum client calls the nurse to her room and states that she knows something awful is going to happen to her. What should the nurse do?

Report this immediately to the health care provider. Explanation: The postpartum woman who develops a pulmonary embolism typically exhibits a sudden onset of dyspnea, pleuritic chest pain and an impending sense of severe apprehension or doom. If the woman experiences any of these signs/symptoms, the nurse will report them immediately to the health care provider.

A pregnant client arrives at the community clinic reporting fever blisters and cold sores on the lips, eyes, and face. The health care provider has diagnosed it as the primary episode of genital herpes simplex virus (HSV), for which antiviral therapy is recommended. Which information should the nurse offer the client when educating her about managing the infection?

Safety of antiviral therapy during pregnancy has not been established. Explanation: The nurse should inform the client that there is no evidence to suggest that antiviral therapy is completely safe during pregnancy. HSV cannot be cured completely, even with timely antiviral drug therapy, and there may be recurrences. The viral shedding process continues for 2 weeks during the primary episode, and kissing during this period may transmit the disease. Recurrent HSV-infection episodes are shorter and milder.

The nurse is assisting with a vaginal birth. The patient is fully dilated, 100% effaced and is pushing. The nurse observes the "turtle sign" with each push and there is no progress. What does the nurse suspect may be occurring with this fetus?

Shoulder dystocia Explanation: The "turtle sign" is the classic sign that alerts the practitioner to the probability of shoulder dystocia. The fetal head delivers, but then retracts similar to a turtle. The fetal head may wiggle from side to side and fail to rotate.

It is necessary for the mother to have a forceps delivery. To reduce complications from this procedure, the nurse should:

empty the mother's bladder. Explanation: Forceps delivery may be outlet, low, or midforceps depending on the station of the fetus and the rotation of the fetal head. Client consent must be obtained and the maternal bladder must be emptied to reduce the chance of bladder injury and to increase the room for the fetus. The anesthesia provider and neonatologist would only be necessary if there was suspicion of complications to the mother and the fetus.

Assessment of a newborn reveals microcephaly. The nurse develops a teaching plan for the parents about the need for follow-up care based on the understanding that the newborn is at risk for developing which complication(s)?

epilepsy cerebral palsy hearing disorders Explanation: Infants with microcephaly are also noted to have additional complications such as epilepsy, cerebral palsy, intellectual disability, and ophthalmologic and hearing disorders. Hydrocephalus and achondroplasia are more commonly seen with macrocephaly.

When caring for a neonate receiving phototherapy, the nurse should remember to:

reposition the neonate frequently. Explanation: Phototherapy works by the chemical interaction between a light source and the bilirubin in the neonate's skin. Therefore, the larger the skin area exposed to light, the more effective the treatment. Changing the neonate's position frequently ensures maximum exposure. Because the neonate will lose water through the skin as a result of evaporation, the amount of formula or water may need to be increased. The neonate is typically undressed to ensure maximum skin exposure. The eyes are covered to protect them from light, and an abbreviated diaper is used to prevent soiling. The skin should be clean and patted dry. Use of lotions would interfere with phototherapy.

In talking to a mother who is 6 hours post-delivery, the mother reports that she has changed her perineal pad twice in the last hour. What question by the nurse would best elicit information needed to determine the mother's status?

"How much blood was on the two pads?" The nurse needs to determine the amount of bleeding the client is experiencing; therefore, the best question to ask the mother is the amount of blood noted on her perineal pads when she changes them. If she had an epidural, she may not feel any pain or discomfort with the bleeding. Although a full bladder can prevent the uterus from contracting, the nurse's main concern is the amount of lochia the mother is having.

The nurse instructs a patient on the use of a vaginal estrogen/progestin rings (NuvaRing) for contraception. Which patient statement indicates that additional instruction is needed?

"I am to take the ring out overnight." Explanation: If the ring is removed for 4 hours for any purpose, it should be replaced with a new ring and a form of barrier protection is to be used for the next 7 days. The ring is not removed overnight. The ring is left in place for 3 weeks and then removed for menstruation during the ring-free week. The ring does not need to be removed for intercourse.

A woman who is about to be discharged after a vaginal birth notices a flea-like rash on her newborn's chest that consists of tiny red lesions all across the nipple line. What is the bestresponse from the nurse when explaining this to the woman?

"It is a normal skin finding in a newborn." Explanation: This most likely is erythema toxicum, also known as newborn rash, and is a common finding that will gradually disappear and not need any treatment. This is often mistaken for staphylococcal pustules. This is not a sign of mistreatment by the woman, nor is it caused by a virus or group B streptococcal (GBS) infection.

The nurse is teaching a pregnant woman with type 1 diabetes about her diet during pregnancy. Which client statement indicates that the nurse's teaching was successful?

"Pregnancy affects insulin production, so I'll need to make adjustments in my diet." Explanation: In pregnancy, placental hormones cause insulin resistance at a level that tends to parallel growth of the fetoplacental unit. Nutritional management focuses on maintaining balanced glucose levels. Thus, the woman will probably need to make adjustments in her diet. Protein needs increase during pregnancy, but this is unrelated to diabetes. Blood glucose monitoring results typically guide therapy.

The student nurse is attending her first cesarean delivery and is asked by the mentor what should be carefully assessed in this infant. After responding "Respiratory status" the student is asked "Why?" What would be the best response?

"There is more fluid present in the lungs at birth after a cesarean delivery than after a vaginal delivery." Explanation: The process of labor stimulates surfactant production, and much of the fetal lung fluid is squeezed out as the fetus moves down the birth canal. This so-called vaginal squeeze is an important way nature helps to clear the airway in preparation for the first breath. The vaginal squeeze also plays a role in stimulating lung expansion. The pressure of the birth canal on the fetal chest releases immediately when the infant is born. The lowered pressure from chest expansion draws air into the lungs.

A woman comes to the clinic. She gave birth about 2 months ago to a healthy term male newborn. During the visit, the woman tells the nurse, "I've noticed that I'm a bit uncomfortable now when we have sexual intercourse. Is there anything that I can do?" The woman's menstrual period has not yet resumed. Which suggestion by the nurse would be most appropriate?

"You might try using a water-soluble lubricant to ease the discomfort." Coital discomfort and localized dryness usually plague most postpartum women until menstruation returns. Water-soluble lubricants can reduce discomfort during intercourse. Although it may take some time for the woman's body to return to its prepregnant state, telling the woman this does not address her concern. Telling her that dyspareunia is normal and that it takes time to resolve also ignores her concern. Kegel exercises are helpful for improving pelvic floor tone but would have no effect on vaginal dryness.

On a routine home visit, the nurse is asking the new mother about her breastfeeding and personal eating habits. How many additional calories should the nurse encourage the new mother to eat daily?

500 additional calories per day The breast-feeding mother's nutritional needs are higher than they were during pregnancy. The mother's diet and nutritional status influence the quantity and quality of breast milk. To meet the needs for milk production, the woman should eat an additional 500 calories per day, 20 grams of protein per day, 400 mg of calcium per day, and 2 to 3 quarts of fluid per day.

At her 16-week checkup, a client's blood pressure is slightly decreased from her prepregnancy level. The nurse evaluates this change based on which statements concerning blood pressure during pregnancy?

A decrease in blood pressure in the second trimester may occur because of placental growth. Explanation: Because the placenta "traps" a great deal of blood for fetal circulation as it expands at about 3 months, maternal blood pressure may temporarily be slightly decreased. Otherwise, blood pressure stays fairly constant throughout pregnancy.

During the assessment, the nurse observes a separation of the rectus muscle that is more than two fingerbreadths wide. Which instruction should the nurse offer the client?

Avoid lifting heavy objects The nurse should teach the client to not lift heavy objects because it could put stress on the abdominal muscles. The client should not be advised to perform regular exercise until the muscles are firmer. Sleeping on a firm mattress or avoiding sleeping on the back does not help the abdominal muscles in any way.

The nurse is caring for a newborn immediately following birth. Which body system is priority for the nurse to monitor during the transition phase?

Cardiopulmonary Explanation: The newborn undergoes numerous changes in the cardiopulmonary system immediately after birth, such as increased blood flow to the lungs, closure of the patent ductus arteriosus, and closure of the foramen ovale. The newborn takes over gas exchange once the umbilical cord is cut. Immunological, integumentary, and thermoregulatory systems are all important pieces of the nursing assessment; however, cardiopulmonary is the priority.

A pregnant client in her 20th week of gestation, having systemic lupus erythematosus (SLE), is admitted to the healthcare unit for management of the exacerbation of SLE. Which of the following would be most appropriate to treat the client's condition?

Corticosteroids Explanation: Corticosteroid therapy is the treatment of choice for pregnant clients with SLE. SLE is an autoimmune disorder where there is deposition of immune complexes in the capillaries and in visceral structure. Corticosteroid therapy in pregnant clients with SLE has a favorable outcome. Hydroxyurea, beta2 agonists and prostaglandin E are not used in the treatment of pregnant clients with SLE. Hydroxyurea is a drug used in the treatment of sickle cell anemia. Beta2 agonists are drugs given to asthmatic clients. Prostaglandin E is a drug used for the induction of labor.

The nurse is preparing a client for discharge and notes an order for rubella vaccine. Which teaching should the nurse prioritize?

Do not to attempt another pregnancy for at least 3 months. The nurse should prioritize the fact that after the immunization, she needs to wait for at least 3 months before attempting to get pregnant again, if desired, so the fetus will not be exposed to the rubella vaccination. The rubella vaccine is a live virus and is considered teratogenic. The other choices are not priorities. Inform the breastfeeding woman that the rubella vaccine crosses over into the breast milk. The newborn benefits from short-term immunity but may become flushed, fussy, or develop a slight rash. Suggest that the woman speak to the pediatrician if she has concerns. The client may also experience a rash, sore throat, headache, and general malaise within 2 to 4 weeks after the injection. The nurse would not advise the new mother that the immunization will prevent hemolytic disease of the infant in her next pregnancy; this is incorrect information.

The nurse is explaining the process of fertilization to a patient who has just learned of being pregnant. On which day during pregnancy should the nurse explain that the embryo implants on the uterine surface?

Eight to 10 days after fertilization Explanation: Implantation, or contact between the growing structure and the uterine endometrium, occurs approximately 8 to 10 days after fertilization. Four days after fertilization the structure is a zygote. Implantation does not occur on the 14th day of a typical menstrual cycle or 10 days after the start of a menstrual flow.

Which assessment finding indicates to the nurse that a newborn has hip subluxation?

Inability of the right hip to abduct Explanation: If the hip joint seems to lock short of this distance of 180 degrees, hip subluxation is suggested. Inward rotation of the right foot, crying when straightening the leg, or drawing the legs underneath when prone does not indicate hip subluxation.

A pregnant patient asks why an α-fetoprotein serum level has been ordered. What should the nurse explain to the patient about this test?

It may reveal chromosomal abnormalities. Explanation: α-Fetoprotein (AFP) is a substance produced by the fetal liver that can be found in both amniotic fluid and maternal serum. The level is abnormally high if the fetus has an open spinal or abdominal wall defect because the open defect allows more AFP to enter the mother's circulation. Although the reason is unclear, the level is low if the fetus has a chromosomal defect such as Down syndrome. Between 85% and 90% of neural tube anomalies, and 80% of Down syndrome babies can be detected by this method. The α-fetoprotein level is not used to screen for placenta functioning, measure fetal liver function, or test the ability of the patient's heart to accommodate the pregnancy.

A pregnant client is admitted to a health care facility with a diagnosis of premature rupture of membranes (PROM). Which of the following tests would the nurse expect to be used to predict fetal lung maturity when the client goes into labor?

Lecithin/sphingomyelin ratio Explanation: The lecithin/sphingomyelin (L/S) ratio of the amniotic fluid helps predict the fetal lung maturity in a client with PROM who goes into labor. A reticulocyte count is used for testing sickle cell anemia. The nitrazine test aids in the diagnosis of PROM and differentiates the amniotic fluid that leaks out after PROM from the normal vaginal secretion. It does not aid in determining fetal lung maturity. A test for antiphospholipids is done in diagnosing antiphospholipids syndrome, and does not aid in determining fetal lung maturity.

The nurse is assisting a woman in scheduling an appointment for a Papanicolaou test. The woman's last menstrual period started on May 2 and ended on May 6. Which date would be most appropriate for the appointment?

May 17 Explanation: A Papanicolaou test should be scheduled at least 1 week after menses ends to increase the chance of getting the best sample of cervical cells without menses. In this case, the date would be May 17.

The nurse is caring for a pregnant client with fallopian tube rupture. Which intervention is the priority for this client?

Monitor the client's vital signs and bleeding. Explanation: A nurse should closely monitor the client's vital signs and bleeding (peritoneal or vaginal) to identify hypovolemic shock that may occur with tubal rupture. Beta-hCG level is monitored to diagnose an ectopic pregnancy or impending abortion. Monitoring the mass with transvaginal ultrasound and determining the size of the mass are done for diagnosing an ectopic pregnancy. Monitoring the FHR does not help to identify hypovolemic shock.

A nurse is assessing a client in her seventh month of pregnancy who has an artificial valve prosthesis. The client is taking an oral anticoagulant to prevent the formation of clots at the valve site. Which of the following nursing interventions is most appropriate in this situation?

Observe the client for signs of petechiae and premature separation of the placenta Explanation: Subclinical bleeding from continuous anticoagulant therapy in the woman has the potential to cause placental dislodgement. Observe a woman who is taking an anticoagulant for signs of petechiae and signs of premature separation of the placenta, therefore, during both pregnancy and labor. The nurse should not urge the client to discontinue the anticoagulant, as this is not within the nurse's scope of practice and, in any case, the client still needs the anticoagulant to prevent clots. Bed rest is prescribed for clients with a thrombus, to prevent it from moving and becoming a pulmonary embolus. Avoiding the use of constrictive knee-high stockings is to prevent thrombus formation.

A female client asks the nurse to explain the purpose of the ovaries besides making eggs for pregnancy. The nurse would explain to the client that the ovaries serve what other purpose(s)?

Ovaries produce estrogen, which helps in the development of secondary sexual characteristics. Ovaries help in the regulation of menstrual cycles through hormonal release. Progesterone produced in the ovaries is necessary for implantation of the fertilized egg and maintenance of the pregnancy. The two female hormones produced by the ovaries are estrogen and progesterone. Estrogen is responsible for development of secondary sex characteristics and progesterone is responsible for implantation of the ova and nourishment of the ova until the placenta takes over the job. Progesterone is secreted by the corpus luteum until around week 12 of the pregnancy, when the placenta takes over production of progesterone.

Which symptom may indicate a complication (not a common discomfort) in the second or third trimester of pregnancy?

Pain underneath the ribs on the right side Explanation: Pain underneath the ribs, especially on the right, may be epigastric pain. Epigastric pain can be a late sign of preeclampsia.

Which medication would you expect to see prescribed for a pregnant woman with an artificial heart valve shortly before or during labor?

Penicillin Explanation: Women with artificial heart valves have an increased risk of subacute bacterial endocarditis following delivery because some bacteria enter the bloodstream from the denuded placental surface. Such bacteria settle in the eddying blood surrounding cardiac shunts or valves.

The nurse is conducting a postpartum examination on a client who reports pain and is unable to sit comfortably. The perineal exam reveals an episiotomy without signs of a hematoma. Which action should the nurse prioritize?

Place an ice pack. The labia and perineum may be bruised and edematous after birth; the use of ice would assist in decreasing the pain and swelling. Applying a warm washcloth would bring more blood as well as fluid to the sore area, thereby increasing the edema and the soreness. Applying a witch hazel pad needs the order of the health care provider. Notifying a health care provider is not necessary at this time as this is considered a normal finding.

After a regular prenatal visit, a pregnant client asks the nurse to describe the differences between placental abruption (abruptio placentae) and placenta previa. Which statement will the nurse include in the teaching?

Placenta previa is an abnormally implanted placenta that is too close to the cervix. Explanation: Placenta previa is a condition of pregnancy in which the placenta is implanted abnormally in the lower part of the uterus and is the most common cause of painless, bright red bleeding in the third trimester. Placental abruption is the premature separation of a normally implanted placenta that pulls away from the wall of the uterus either during pregnancy or before the end of labor. Placental abruption can result in concealed or apparent dark red bleeding and is painful. Immediate intervention is required for placental abruption.

A patient having an examination to check the placement of an intrauterine device (IUD) is diagnosed as being pregnant. For which action should the nurse prepare the patient at this time?

Removal of the IUD Explanation: A patient may become pregnant with an intrauterine device (IUD) in place. If this occurs, it needs to be removed to prevent infection during pregnancy. The fetus does not need to be aborted, and the patient will not spontaneously abort because the IUD is in place. The IUD cannot remain in place because of the risk for infection.

The primigravida client is surprised by the continued uterine contractions while holding her new baby. Which explanation by the nurse will best explain these contractions?

Seals off the blood vessels at the site of the placenta The contractions of the uterus help to constrict the vessels where the placenta was located. This does decrease the flow of blood but is secondary in occurrence to the constriction of the blood vessels. Uterine contraction also leads to uterine involution, which normally occurs at a predictable rate. Uterine involution assists in closing the cervix. Again, the other options are secondary to the constriction of blood vessels at the placental site.

A nurse is assessing a newborn and notifies the primary health care provider because the nurse suspects increased intracranial pressure. When reporting the findings, which of the following would the nurse most likely include?

Seizure activity Explanation: Seizure activity is a change in neurologic status and can indicate increased intracranial pressure. Overriding sutures; soft, flat (nonbulging) fontanels; and normal vital signs are normal newborn findings.

A patient is concerned about the mercury levels in fish and asks the nurse which fish are safe to eat. The best response is

Shrimp, canned tuna, pollack, and catfish Explanation: The larger the fish, the higher the concentration of mercury will be in that fish. Fish such as shrimp, catfish, anchovies, and sardines are small and therefore have small amounts of mercury.

A woman who is two days postpartum has painful hemorrhoids. Which of the following positions would you suggest she use for resting?

Sims' position A Sims' position relieves pressure on rectal veins, thus reducing the size and pain of hemorrhoids. A knee-chest position is not recommended.

The nurse is teaching a client about mastitis. Which statement should the nurse include in her teaching?

Symptoms include fever, chills, malaise, and localized breast tenderness. Explanation: Mastitis is an infection of the breast characterized by flu-like symptoms, along with redness and tenderness in the breast. The most common causative agent is Staphylococcus aureus. Breast abscess is rarely a complication of mastitis if the client continues to empty the affected breast. Mastitis usually occurs in one breast, not bilaterally.

Which of the following is true regarding the newborn's fontanelles?

The anterior fontanelle is diamond shaped and measures about 3.5 cm. The posterior fontanelle is triangular shaped and measures about 1 cm.

When assessing the postpartum client 2 hours after giving birth, which finding indicates the need for further action?

The fundus is firm and deviated sharply to the right side of the abdomen. In the immediate postpartum period, the fundus is regularly assessed. The fundus must be firm. A boggy fundus indicates uterine atony and will result in blood loss. The fundus is to be midline in the abdomen. A deviation to the side may indicate a full bladder. In the immediate hours after birth, the fundus may be found at one fingerbreadth above or below the umbilicus.

A preterm newborn has just received synthetic surfactant through an endotracheal tube by a syringe. Which intervention should the nurse implement at this point?

Tip the infant into an upright position. Explanation: It's important the infant is tipped to an upright position following administration of surfactant and the infant's airway is not suctioned for as long a period as possible after administration of surfactant to help it reach lower lung areas and avoid suctioning the drug away. A blood sample may be taken to rule out a streptococcal infection, which mimics the signs of RDS, but this would have been done before administration of surfactant. The infant should not be placed supine in a radiant heat warmer at this time but should be held in an upright position.

Assessment of a pregnant client reveals that she is experiencing Braxton -Hicks contractions. Which of the following would the nurse explain as the cause of these contractions?

Uterine distension Explanation: The nurse should explain that Braxton-Hicks contractions occur due to uterine distension. Braxton-Hicks contractions occur throughout pregnancy as painless uterine contractions. As labor approaches it is associated with low backache. Increase in the estrogen levels also causes Braxton-Hicks contractions. Fall in the estrogen level, release of prostaglandins and catecholamines do not cause Braxton-Hicks contraction.

A client is experiencing postpartum hemorrhage and the nurse begins to massage her fundus. Which action would be most appropriate for the nurse to do when massaging the woman's fundus?

Wait until the uterus is firm to express clots. Explanation: The uterus must be firm before attempts to express clots are made because application of firm pressure on an uncontracted uterus could lead to uterine inversion. One hand is placed on the fundus and the other hand is placed on the area above the symphysis pubis. Circular motions are used for massage. There is no specified amount of time for fundal massage. Uterine tissue responds quickly to touch, so it is important not to overmassage the fundus.

A postpartum client is recovering from the birth and emergent repair of a cervical laceration. Which sign on assessment should the nurse prioritize and report to the health care provider?

Weak and rapid pulse Explanation: Excessive hemorrhage puts the client at risk for hypovolemic shock. Signs of impending shock include a weak and rapid pulse, decreased blood pressure, tachypnea, and cool and clammy skin. These findings should be reported immediately to the health care provider so that proper intervention for the client may be instituted.

The nurse is preparing discharge for a client who plans to bottle-feed her infant. Which instruction should the nurse prioritize for this client in the discharge teaching?

Wear a tight, supportive bra. The client trying to dry up her milk supply should do as little stimulation to the breast as possible. She needs to wear a tight, supportive bra and use ice. Running warm water over the breasts in the shower will only stimulate the secretion, and therefore the production, of milk. Massaging the breasts will stimulate them to expel the milk and therefore produce more milk, as will expressing small amounts of milk when the breasts are full.

Disseminated intravascular coagulation is a life-threatening condition that the nurse recognizes can occur as a complication secondary to which primary conditions?

abruptio placenta severe preeclampsia septicemia Explanation: DIC is not itself a specific illness; rather it is always a secondary diagnosis that occurs as a complication of placental abruption, anaphylactoid syndrome of pregnancy, intrauterine fetal death with prolonged retention of the fetus, acute fatty liver of pregnancy, severe preeclampsia, HELLP syndrome (hemolysis, i.e., the breakdown of red blood cells, elevated liver enzymes, and low platelet count), septicemia, and postpartum hemorrhage.

A pregnant woman with a history of mitral valve stenosis is to be prescribed medication as treatment. Which medication class would the nurse expect the client to be prescribed?

anticoagulant Explanation: In mitral valve stenosis, it is difficult for blood to leave the left atrium. A secondary problem of thrombus formation may develop as a result of noncirculating blood. A woman may need to be prescribed an anticoagulant to prevent this complication. Vasodilators are used for peripartum cardiomyopathy. Inotropics are used for heart failure. Angiotensin receptor blockers are used for congestive heart failure.

Seven hours ago, a multigravida woman gave birth to a male infant weighing 4,133 g. She has voided once and calls for a nurse to check because she states that she feels "really wet" now. Upon examination, her perineal pad is saturated. The immediate nursing action is to:

assess and massage the fundus. This woman is a multigravida who gave birth to a large baby and is at risk for hemorrhage. The other actions are to be done after the initial fundal massage.

A nurse is caring for a pregnant client with heart disease in a labor unit. Which intervention is most important in the first 48 hours postpartum?

assessing for cardiac decompensation Explanation: The nurse should assess the client with heart disease for cardiac decompensation (s/s indicating that the heart is no longer able to maintain efficient circulation, which is most common from 28 to 32 weeks of gestation and in the first 48 hours postpartum. Limiting sodium intake, inspecting the extremities for edema, and ensuring that the client consumes a high-fiber diet are interventions during pregnancy not in the first 48 hours postpartum.

A nurse finds the uterus of a postpartum woman to be boggy and somewhat relaxed. This a sign of which condition?

atony The uterus in a postpartum client should be midline and firm. A boggy or relaxed uterus signifies uterine atony, which can predispose the woman to hemorrhage.

A nurse is teaching a 30-year-old gravida 1 who has sickle cell anemia. Providing education on which topic is the highest nursing priority?

avoidance of infection Explanation: Prevention of crises, if possible, is the focus of treatment for the pregnant woman with sickle cell anemia. Maintaining adequate hydration, avoiding infection, getting adequate rest, and eating a balanced diet are all common-sense strategies that decrease the risk of a crisis. Fat intake does not need to be decreased and immunoglobulins are not normally administered. Constipation is not usually a result of sickle cell anemia.

During a prenatal visit a pregnant client asks the nurse how to tell whether the contractions she is having are true contractions or Braxton Hicks contractions. Which description should the nurse mention as characteristic of true contractions?

begin irregularly but become regular and predictable felt first in lower back and sweep around to the abdomen in a wave increase in duration, frequency, and intensity Explanation: True contractions begin irregularly but become regular and predictable; are felt first in the lower back and sweep around to the abdomen in a wave; continue no matter what the woman's level of activity; increase in duration, frequency, and intensity; and achieve cervical dilatation. False (Braxton Hicks) contractions begin and remain irregular; are felt first abdominally and remain confined to the abdomen and groin; often disappear with ambulation or sleep; do not increase in duration, frequency, or intensity; and do not achieve cervical dilatation.

A nurse is caring for a client who has had an intrauterine fetal death with prolonged retention of the fetus. For which signs and symptoms should the nurse watch to assess for an increased risk of disseminated intravascular coagulation?

bleeding gums tachycardia acute renal failure Explanation: The nurse should monitor for bleeding gums, tachycardia, and acute renal failure to assess for an increased risk of disseminated intravascular coagulation in the client. The other clinical manifestations of this condition include petechiae, ecchymosis, and uncontrolled bleeding during birth. Hypotension and amount of lochia greater than usual are findings that might suggest a coagulopathy or hypovolemic shock.

While performing a physical assessment of a newborn boy, the nurse notes diffuse edema of the soft tissues of his scalp that crosses suture lines. The nurse documents this finding as:

caput succedaneum. Caput succedaneum is localized edema on the scalp, a poorly demarcated soft tissue swelling that crosses the suture lines. Molding refers to the elongated shape of the fetal head as it accommodates to the passage through the birth canal. Microcephaly refers to a head circumference that is 2 standard deviations below average or less than 10% of normal parameters for gestational age. Cephalohematoma is a localized effusion of blood beneath the periosteum of the skull.

A client with a family history of cervical cancer is to undergo a Papanicolaou test. During the client education, what group should the nurse include as at risk for cervical cancer?

clients who have genital warts Explanation: The presence of genital warts (condyloma) increases the risk of developing cervical cancer. Women with metrorrhagia or irregular menstrual cycles are at an increased risk of developing breast cancer, not cervical cancer. Clients who have never had a baby or those with a history of fibrocystic breast disease have an increased risk of developing breast cancer, but not cervical cancer.

A pregnant woman with diabetes at 10 weeks' gestation has a glycosylated hemoglobin (HbA1c) level of 13%. At this time the nurse should be most concerned about which possible fetal outcome?

congenital anomalies Explanation: A HbA1c level of 13% indicates poor glucose control. This, in conjunction with the woman being in the first trimester, increases the risk for congenital anomalies in the fetus. Elevated glucose levels are not associated with incompetent cervix, placenta previa, or placental abruption (abruptio placentae).

Which finding would lead the nurse to suspect that the fetus of a woman in labor is in hypertonic uterine dysfunction?

contractions most forceful in the middle of uterus rather than the fundus Explanation: Contractions that are more forceful in the midsection of the uterus rather than in the fundus suggest hypertonic uterine dysfunction. Reports of severe back pain are associated with a persistent occiput posterior position due to the pressure of the fetal head on the woman's sacrum and coccyx. Cervical dilation (dilatation) that has not progressed past 2 cm is associated with dysfunctional labor. A breech position is one in which the fetal presenting part is the buttocks or feet.

The nurse notices while holding him upright that a day-old newborn has a significantly indented anterior fontanelle. She immediately brings it to the attention of the primary care provider. What does this finding indicate?

dehydration Explanation: The anterior fontanelle can be felt as a soft spot. It should not appear indented (a sign of dehydration) or bulging (a sign of increased intracranial pressure) when the infant is held upright. Vernix caseosa is the white, cream cheese-like substance that serves as a skin lubricant in utero. Some of it is invariably noticeable on a term newborn's skin, at least in the skin folds, at birth. Cyanosis is a condition of decreased oxygenation that results in the skin having a blue hue.

The nurse, assessing the lochia of a client, attempts to separate a clot and identifies the presence of tissue. Which observation would indicate the presence of tissue?

difficult to separate clots If tissue is identified in the lochia, it is difficult to separate clots. Yellowish-white lochia indicates increased leukocytes and decreased fluid content. Easily separable lochia indicates the presence of clots only. Foul-smelling lochia indicates endometritis.

Which effect would the nurse identify as a normal physiologic change in the renal system due to pregnancy?

dilation of the renal pelvis Explanation: The renal pelvis becomes dilated during pregnancy, possibly due to the effect of progesterone on smooth muscle. The glomerular filtration rate increases during pregnancy. The kidneys enlarge during pregnancy. The ureters elongate, widen, and become more curved above the pelvic rim.

A nurse is conducting a discussion group at a local women's health center about violence and women. The nurse would include that women are at a higher risk for violence at which time?

during pregnancy Explanation: Women are at a higher risk for violence during pregnancy. Recent research findings indicate that having children does not protect women. They are more vulnerable during this time, and the abusers take advantage of it.

A nurse is reviewing the medical record of a client. Which finding would lead the nurse to suspect that the client is experiencing polycystic ovarian syndrome?

elevated blood insulin levels anovulation triglyceride level of 175 mg/dL Explanation: Polycystic ovarian syndrome is a multifaceted disorder, and central to its pathogenesis are hyperandrogenemia and hyperinsulinemia. PCOS is associated with obesity, hyperinsulinemia, elevated luteinizing hormone levels (linked to ovulation), elevated androgen levels (virilization), hirsutism (male-pattern hair growth), follicular atresia (ovarian growth failure), ovarian growth and cyst formation, anovulation (failure to ovulate), infertility, type 2 diabetes, sleep apnea, amenorrhea (absence of menstruation or irregular periods) and metabolic syndrome, which is characterized by abdominal obesity (waist circumference >35 in.), dyslipidemia (triglyceride level >150 mg/dL, high-density lipoprotein cholesterol [HDL-C] level <50 mg/dL), elevated blood pressure, a pro-inflammatory state characterized by an elevated C-reactive protein level, and a prothrombotic state characterized by elevated PAI-1 and fibrinogen levels.

A woman with an incomplete abortion is to receive misoprostol. The woman asks the nurse, "Why am I getting this drug?" The nurse responds to the client, integrating understanding that this drug achieves which effect?

ensures passage of all the products of conception Explanation: Misoprostol is used to stimulate uterine contractions and evacuate the uterus after an abortion to ensure passage of all the products of conception. Rho(D) immune globulin is used to suppress the immune response and prevent isoimmunization.

A nurse is caring for a pregnant client who is in the active phase of labor. At what interval should the nurse monitor the client's vital signs?

every 30 minutes Explanation: When a pregnant client is in the active phase of labor, the nurse should monitor the vital signs every 30 minutes. The nurse should monitor the vital signs every 30 to 60 minutes if the client is in the latent phase of labor and every 15 to 30 minutes during the transition phase of labor. Temperature is usually monitored every 4 hours in the active phase of labor.

The nurse is conducting an annual examination on a young female who reports her last menses was 2 months ago. The client insists she is not pregnant due to a negative home pregnancy test. Which assessment should the nurse use to assess confirm the pregnancy?

fetal heartbeat Explanation: The only positive sign of pregnancy is a sign or symptom that could only be attributable to the fetus; thus, fetal heartbeat can have no other origin. Chadwick sign is a color change in the cervix, vagina, and perineum; these could all be the result of other causes. A positive urine hCG is a probable sign as it can be related to causes other than pregnancy. A change in the size and shape of the uterus can occur due to other causes.

A nurse is teaching a group of nursing students about the mechanism of labor when the fetus is in a cephalic presentation. The nurse determines the session is successful when the students correctly place the following events in which order?

flexion internal rotation extension external rotation expulsion Explanation: The sequence of normal mechanism of labor involves descent, flexion, internal rotation, extension, external rotation, and expulsion. Following descent, the baby's head encounters resistance and flexes so that the chin touches the chest. During internal rotation, the occiput is rotated 45° anteriorly so as to lie beneath the symphysis pubis. The baby's neck is twisted in internal rotation. Following internal rotation, the delivery of the head is by extension. Once the head is born by extension, there is external rotation, in which it turns 45° more so that the shoulders lie anterior posteriorly.

A client has been admitted with placental abruption. She has lost 1,200 mL of blood, is normotensive, and ultrasound indicates approximately 30% separation. The nurse documents this as which classification of abruptio placentae?

grade 2 Explanation: The classifications for abruptio placentae are: grade 1 (mild) - minimal bleeding (less than 500 mL), 10% to 20% separation, tender uterus, no coagulopathy, signs of shock or fetal distress; grade 2 (moderate) - moderate bleeding (1,000 to 1,500 mL), 20% to 50% separation, continuous abdominal pain, mild shock, normal maternal blood pressure, maternal tachycardia; grade 3 (severe) - absent to moderate bleeding (more than 1,500 mL), more than 50% separation, profound shock, dark vaginal bleeding, agonizing abdominal pain, decreased blood pressure, significant tachycardia, and development of disseminated intravascular coagulopathy. There is no grade 4.

While assessing a newborn, the nurse notes that half the body appears red while the other half appears pale. The nurse interprets this finding as:

harlequin sign. Explanation: Harlequin sign refers to the dilation of blood vessels on only one side of the body. It gives a distinct midline demarcation, which is pale on one side and red on the opposite. Stork bites are superficial vascular areas found on the nape of the neck, eyelids, between the eyes and upper lip. Mongolian spots are blue or purple splotches that appear on the lower back and buttocks. Erythema toxicum is a benign, idiopathic, generalized, transient rash that resembles flea bites.

An expectant mother is on heparin for previous blood clots and voicing concerns about how her medications will affect her baby. The nurse would inform the mother that:

heparin does not cross the placenta and is safe for her to take. Explanation: Heparin is a medication that does not cross the placenta and therefore is safe to use during pregnancy. Not all medications cause fetal sequelae. It is not recommended to abruptly discontinue any medication without consulting the mother's health care provider and heparin is the safest anticoagulant for a pregnant woman to take.

A nurse is analyzing a journal article that explains the changes at birth from fetal to newborn circulation. The nurse can point out the closure of the ductus arteriosus is related to which event after completing the article?

higher oxygen content of the circulating blood Explanation: The first few breaths greatly increase the oxygen content of circulating blood. This chemical change (i.e., higher oxygen content of the blood) contributes to the closing of the ductus arteriosus, which eventually becomes a ligament. A drop in the pressure results in a reversal of pressures in the right and left atria, causing the foramen ovale to close, which redirects blood to the lungs. A drop in blood pressure and higher oxygen levels at the respiratory centers of the brain do not result in the closure of the foramen ovale.

An obese woman with diabetes has just given birth to a term, large-for-gestational-age (LGA) newborn. Which condition should the nurse most expect to find in this infant?

hypoglycemia Explanation: LGA infants also need to be carefully assessed for hypoglycemia in the early hours of life because large infants require large amounts of nutritional stores to sustain their weight. If the mother had diabetes that was poorly controlled (the cause of the large size), the infant would have had an increased blood glucose level in utero to match the mother's; this caused the infant to produce elevated levels of insulin. After birth, these increased insulin levels will continue for up to 24 hours of life, possibly causing rebound hypoglycemia.

A client appears to be resting comfortably 12 hours after giving birth to her first child. In contrast, she labored for more than 24 hours, the primary care provider had to use forceps to deliver the baby, and she had multiple vaginal examinations during labor. Based on this information what postpartum complication is the client at risk for developing?

infection Explanation: There are many risk factors for developing a postpartum infection: operative procedures (e.g., forceps, cesarean section, vacuum extraction), history of diabetes, prolonged labor (longer than 24 hours), use of Foley catheter, anemia, multiple vaginal examinations during labor, prolonged rupture of membranes, manual extraction of placenta, and HIV.

Which stage of syphilis occurs when the infected person has no signs or symptoms of syphilis?

latency Explanation: Primary syphilis occurs 2 to 3 weeks after initial inoculation with the organism. Secondary syphilis occurs when the hematogenous spread of organisms from the original chancre leads to generalized infection. A period of latency occurs when the infected person has no signs or symptoms of syphilis. Tertiary syphilis presents as a slowly progressive inflammatory disease with the potential to affect multiple organs.

A 35-year-old client has just given birth to a healthy newborn during her 43rd week of gestation. What should the nurse expect when assessing the condition of the newborn?

meconium aspiration in utero or at birth Explanation: Infants born after 42 weeks of pregnancy are postterm. These infants are at a higher risk of swallowing or aspirating meconium in utero or after birth. As soon as the infant is born, the nurse usually suctions out the secretions and fluids in the newborn's mouth and throat before the first breath to avoid aspiration of meconium and amniotic fluid into the lungs. Seizures, respiratory distress, cyanosis, and shrill cry are signs and symptoms of infants with intracranial hemorrhage. Intracranial hemorrhage can be a dangerous birth injury that is primarily a problem for preterm newborns, not postterm neonates. Yellow appearance of the newborn's skin is usually seen in infants with jaundice. Tremors, irritability, high-pitched or weak cry, and eye rolling are seen in infants with hypoglycemia.

The newborn nursery nurse is admitting a small-for-gestational-age (SGA) infant and is reviewing the maternal history. What factor in the maternal history would the nurse correlate as a risk factor for a SGA infant?

placental factors Explanation: Assessment of the SGA infant begins by reviewing the maternal history to identify risk factors such as placental factors with abnormal umbilical cord insertion, chronic placental abruption, malformed and smaller placentas, with placental previa or placental insufficiency being the main placental causes. Blood group incompatibility, having many pregnancies, and being over the age of 30 will not cause an SGA infant.

A patient who experiences premature rupture of membranes can expect to be put on pelvic rest. The nurse should explain to the patient that pelvic rest involves which of the following?

placing nothing in the vagina Explanation: Pelvic rest is a situation in which nothing is placed into the vagina (including tampons and the practitioner's fingers to perform a cervical examination). The other options (strict bed rest and bed rest with bathroom privileges) also might be ordered when a woman has premature rupture of membranes.

A couple who is in for fertility testing ask the nurse what tests are commonly performed to assess fertility. The nurse replies that there are only three primary tests that are used. What are these tests?

semen analysis, ovulation monitoring, and tubal patency assessment Explanation: Only three tests are commonly used to test fertility: semen analysis in the male and ovulation monitoring and tubal patency assessment in the female. Additional testing may be performed both on the man and the woman, which may include urinalysis, serologic test for syphilis, and pelvic sonogram, but these are not the primary tests used.

A nurse is caring for a baby girl born at 34 weeks' gestation. Which feature should the nurse identify as those of a preterm newborn?

shiny heels and palms Explanation: A preterm newborn has shiny heels and palms with few creases. The eyelids of the preterm newborn are edematous, and not paper-thin. The external genitalia in the preterm baby girl appear large with widely spaced labia, and not closely approximated. Vernix is scant in postterm newborns and is excessive in premature infants.

A nurse is conducting a presentation at a community health center about congenital malformations. The nurse describes that some common congenital malformations can occur and are recognized to be caused by multiple genetic and environmental factors. Which example would the nurse most likely cite?

spina bifida Explanation: Spina bifida is a multifactorial inherited disorder thought to be due to multiple genetic and environmental factors. Cystic fibrosis is considered an autosomal recessive inherited disorder, while color blindness and hemophilia are considered X-linked inheritance disorders.

The nurse educates the vegetarian client about which nutritional need during pregnancy?

taking a B12 supplement Explanation: B12 is found almost exclusively in animal proteins and therefore is absent in the vegetarian diet. Fiber and dark green vegetables are needed. Vitamins A and C are not protein based and are found in a vegetarian diet.

A fetus is capable of producing antibodies. The finding of IgM antibodies in an infant at birth implies that:

the fetus contracted an infection during intrauterine life. Explanation: Because IgM antibodies are too large to cross the placenta, the only way they could be present in a fetus is if they were produced by the fetus in response to invading antigens (Toxoplasma gondii, which causes toxoplasmosis).

A nurse is speaking to a local women's group about the various types of cancer affecting the female reproductive tract. The nurse explains that ovarian cancer is the leading cause of death from gynecologic malignancies based on the understanding that this type of cancer:

typically manifests with vague symptoms resulting in late diagnosis. Explanation: Tumors of the ovary have been lethal largely because they present with nonspecific symptoms and therefore frequently are far advanced and inoperable by the time they are diagnosed. Ease of spread and types of cells involved are not reasons underlying the fatal nature of this type of cancer. Ovarian cancer is not associated with sexually transmitted infections. Cervical cancer is linked to human papillomavirus infection.

A nurse is assessing a client diagnosed with bacterial vaginosis. What is a symptom of bacterial vaginosis?

vaginal odor smelling of fish Explanation: Bacterial vaginosis causes a "stale fish" vaginal odor. Cottage cheese-like discharge is seen in candidiasis and not in bacterial vaginosis, in which the discharge is thin and gray-white. Intense itching of the vulva is associated with candidiasis, not vaginosis. Warts in the vulva and labia are seen in human papillomavirus infection, not in vaginosis.

A nurse is interviewing a couple at a preconception counseling session. The couple is of Greek heritage and are concerned about the possibility of their children being born with a genetic disorder. Based on the nurse's understanding of genetic disorders, the nurse would identify this couple as being at risk for which condition?

β-thalassemia Explanation: Different ancestry backgrounds cause different genetic disorders to be more common in some ethnic groups than in others. The blood disorder β-thalassemia, for example, occurs most frequently in families of Greek or Mediterranean heritage, whereas α-thalassemia occurs most often in persons from the Philippines or southeast Asia. Sickle-cell anemia occurs most often in people with an African ancestry. Tay-Sachs disease, a deterioration of muscle and mental facilities, occurs most often in people of eastern Jewish ancestry.

A client who is HIV-positive is in her second trimester and remains asymptomatic. She voices concern about her newborn's risk for the infection. Which statement by the nurse would be mostappropriate?

"Antiretroviral medications are available to help reduce the risk of transmission." Explanation: Drug therapy is the mainstay of treatment for pregnant women infected with HIV. The goal of therapy is to reduce the viral load as much as possible; this reduces the risk of transmission to the fetus. Decisions about the method of birth should be based on the woman's viral load, duration of ruptured membranes, progress of labor, and other pertinent clinical factors. The newborn is at risk for HIV because of potential perinatal transmission. Waiting until after the infant is born may be too late.

Which statement made by a new nurse indicates additional teaching is needed on the topic of hyperbilirubinemia (physiologic jaundice) in newborns?

"Breastfed babies need supplements of glucose water to help lower bilirubin levels." Physiologic jaundice (hyperbilirubinemia) is characterized by a yellowish skin, mucous membranes, and sclera that occurs within the first 3 days of life. Physiologic jaundice is caused by accelerated destruction of fetal RBCs that have a shortened life span (80 days compared with the adult 120 days). Normally the liver removes bilirubin (the by-product of RBC destruction) from the blood and changes it into a form that can be excreted. As the red blood cell breakdown continues at a fast pace, the newborn's liver cannot keep up with bilirubin removal. Thus, bilirubin accumulates in the blood, causing the characteristic signs of physiologic jaundice. Expose the newborn to natural sunlight for short periods of time throughout the day to help oxidize the bilirubin deposits on the skin. Glucose water supplementation should be avoided since it hinders elimination.

Your patient is very conscious of the weight she gained during her pregnancy. She has decided to breastfeed her baby and asks you how many calories a day extra she should be eating so that she and her baby are healthy. She states she does not want to gain any extra weight from over-eating. What would be your best response?

"You should be eating an extra 200 kcal over what you were eating while you were pregnant." Explanation: Instruct the woman who is not breastfeeding to decrease her caloric intake by approximately 300 kcal per day (i.e., she should reduce her intake to prepregnancy levels). The lactating woman will need to add an additional 200 kcal above the pregnancy requirement of 300 kcal per day, for a total of 500 kcal above prepregnancy requirements.

A client with asthma is confused by her primary care provider continuing her medication while she is pregnant, since she read online the medications can cause birth defects. What is the nurse's best response?

"Your primary care provider will order safe doses of your medication." Explanation: Women should take no medication during pregnancy except that prescribed by their primary care provider. The PCP will work with the mother to ensure the safest amount is given to adequately handle the mother's health issues and not injure the fetus. The PCP must weigh the risks against the benefits for both the mother and her fetus. The nurse should not encourage the client to stop her asthma medication as that may result in the client having an asthma attack, which could result in injury to the fetus or even miscarriage. The nurse should not tell the client a drug will not cause any defects, especially if it is known that it can. That could make the nurse liable for damages. The nurse should inform the PCP of the client's concerns; however, it is more important for the nurse to calm the client's anxiety and offer positive reinforcement that the PCP is working hard to protect the mother and infant from harm.

A patient in labor who is dilated 7 cm reports that narcotic pain medication given 3 hours ago has worn off and is asking for another dose. How should the nurse respond to this request?

"Your stage of labor makes giving another dose unsafe." Explanation: The timing of administration of narcotics in labor is especially important. If given close to birth, because the fetal liver takes 2 to 3 hours to activate a drug, the effect will not be registered in the fetus for 2 to 3 hours after maternal administration. For this reason, narcotics are preferably given when the mother is more than 3 hours away from birth. This allows the peak action of the drug in the fetus to have passed by the time of birth. The nurse does not need to get permission from the physician. Pain medication can be provided when needed and not on a set schedule of every 4 hours. The patient is nearing delivery so 3 hours from the last dose will not influence the decision to provide more medication.

A client is admitted to the unit in preterm labor. In preparing the client for tocolytic drug therapy, the nurse anticipates that the client's pregnancy may be prolonged for how long when this therapy is used?

2 to 7 days Explanation: Tocolytic drugs may prolong the pregnancy for 2 to 7 days. During this time, steroids can be given to improve fetal lung maturity, and the woman can be transported to a tertiary care center.

A pregnant patient is diagnosed with placenta previa. Which action should the nurse implement immediately for this patient?

Assess fetal heart sounds with an external monitor. Explanation: For placenta previa, the nurse should attach external monitoring equipment to record fetal heart sounds and uterine contractions. Internal pressure gauges to measure uterine contractions are contraindicated. A pelvic or rectal examination should never be done with painless bleeding late in pregnancy because any agitation of the cervix when there is a placenta previa might tear the placenta further and initiate massive hemorrhage, which could be fatal to both mother and child. To ensure an adequate blood supply to the patient and fetus, the patient should be placed immediately on bed rest in a side-lying position.

Upon assessment, the nurse notes a postpartum client has increased vaginal bleeding. The client had a forceps birth that resulted in lacerations 4 hours ago. What should the nurse do next?

Assess for uterine contractions. The nurse needs to identify whether the bleeding is from lacerations or uterine atony. This can be done by looking for a well contracted uterus with bright-red vaginal bleeding. Lacerations commonly occur during forceps birth. In subinvolution of the uterus, there is inadequate contraction, resulting in bleeding. A boggy uterus with vaginal bleeding is seen in uterine atony. Once the nurse knows the cause of the bleeding, the condition can be treated.

A nurse is caring for an infant. A serum blood sugar of 40 was noted at birth. What care should the nurse provide to this newborn?

Begin early feedings either by the breast or bottle. Explanation: The nurse should provide some nutrition to any infant born with hypoglycemia. Dextrose should be given intravenously only if the infant refuses oral feedings, not before offering the infant oral feedings. Placing the infant on a radiant warmer will not help maintain blood glucose levels. The nurse should focus on decreasing blood viscosity in an infant who is at risk for polycythemia, not hypoglycemia.

Which strategies is the nurse correct to utilize when attempting to awaken a potentially sleeping fetus?

Clap near the mother's abdomen Provide the mother a cold beverage Place hands on the abdomen to move the fetus Use vibroacoustic stimulation Explanation: The nurse is correct to arouse the fetus in a variety of ways. The nurse can use audio stimulation such as clapping near the abdomen or using vibroacoustic stimulation. Providing the mother a cold beverage can also arouse the fetus. Feeling the mother's abdomen for the location of the fetus and moving the body parts can also cause the fetus to move and/or kick. Simply laying the mother on her side may cause a shift in the fetus but is not always enough to arouse the fetus.

A client is Rh-negative and has given birth to her newborn. What should the nurse do next?

Determine the newborn's blood type and rhesus. The nurse first needs to determine the rhesus of the newborn to know if the client needs Rh immunoglobulins. Mothers who are Rh-negative and have given birth to an infant who is Rh-positive should receive an injection of Rh immunoglobulin within 72 hours after birth; this prevents a sensitization reaction to Rh-positive blood cells received during the birthing process. Women should receive the injection regardless of how many children they have had in the past.

The nurse discovers a soft systolic murmur when auscultating the heart of a client at 32 weeks' gestation. Which action would be most appropriate?

Document this and continue to monitor the murmur at future visits. Explanation: Due to the increased blood volume that occurs with pregnancy, soft systolic murmurs may be heard and are considered normal.

A pregnant woman comes to the clinic for a prenatal visit for her third pregnancy. She reveals she had a previous miscarriage at 12 weeks and her 3-year-old son was born at 32 weeks. How should the nurse document this woman's obstetric history?

G3, T0, P1, A1, L1 The woman's obstetric history would be documented as G3, T0, P1, A1, L1. G (gravida) = 3 (past and current pregnancy), T (term pregnancies) = 0, P (number of preterm pregnancies) = 1, A (number of pregnancies ending before 20 weeks viability to include miscarriage) = 1, and L (number of living children) = 1.

A nurse is reviewing a journal article about fetal development and the formation of various body systems. When reading about the development of the digestive system, the nurse finds information related to the developmental abnormality of omphalocele. The nurse demonstrates understanding of this information by identifying which situation as the reason for an omphalocele?

Intestines failed to return to the abdominal cavity during intrauterine life. Explanation: Because intestines grow more rapidly than the abdominal cavity, they are pushed into the base of the cord for a short span during intrauterine life. If for some reason they do not return to the abdominal cavity, an omphalocele will result.

A newborn is suspected to have fetal alcohol syndrome as a result of maternal use of alcohol during pregnancy. Which of the following would the nurse expect to assess.

Low nasal bridge Explanation: A low nasal bridge is seen in fetal alcohol syndrome (FAS). The other features of FAS include thin, flat upper lip, small eyes with short palpebral fissure, flattened midface with a short nose. The facial features result from damage to the embryonic cells in early pregnancy.

After delivery, a patient is diagnosed with placenta succenturiata. For what procedure should the nurse prepare this patient?

Manual removal of accessory lobes Explanation: A placenta succenturiata is a placenta that has one or more accessory lobes connected to the main placenta by blood vessels. This disorder needs to be recognized because the small lobes may be retained in the uterus after birth, leading to severe maternal hemorrhage. Once the remaining lobes are recognized and removed from the uterus manually, the uterus will contract with no adverse maternal effects. Uterine lavage is not a treatment for this disorder. This disorder is not specifically associated with an episiotomy. No fetal abnormality is associated with this disorder.

A 37-year-old primigravida client at 40 weeks' gestation is seen in the clinic for a scheduled prenatal visit. What report by the client would lead the nurse to predict the woman is close to labor?

Nesting Explanation: Nesting is the activity or burst of energy women often experience prior to the onset of labor. The client could express feeling energetic or through a report of her activities at home. Dilation, effacement, and cervical ripening are all observed by the provider during assessment.

Which nursing interventions during labor help the nurse identify a breech presentation in the fetus?

Palpating the fetal head in the fundus during Leopold maneuvers Locating fetal heart tones slightly above the umbilicus Noting the passage of meconium after the rupture of membranes Explanation: During labor, palpating the head in the fundus during Leopold maneuver, locating fetal heart tones slightly above the umbilicus, feeling the buttocks during a vaginal examination, and noting the passage of meconium after rupture of the membranes help the nurse identify a breech presentation (if the same has not been identified earlier). Checking for Bandl's ring formation after the rupture of membranes and observing for a fetal body depression such as the neck or abdomen suggest pathologic retractions and constriction rings.

An 18-year-old client has given birth at 28 weeks' gestation and her newborn is showing signs of respiratory distress syndrome (RDS). Which statement is true for a newborn with RDS?

RDS is caused by a lack of alveolar surfactant. Explanation: Respiratory distress syndrome (RDS) is a serious breathing disorder caused by a lack of alveolar surfactant. Betamethasone, a glucocorticoid, is often given to the mother 12 to 24 hours before a preterm birth to help reduce the severity of RDS, not to the newborn following birth. Respiratory symptoms in the newborn with RDS typically worsen, not improve, within a short period of time after birth. Diagnosis of RDS is made based on a chest X-ray and the clinical symptoms of increasing respiratory distress, crackles, generalized cyanosis, and heart rates exceeding 150 beats per minute (not below 50 beats per minute).

The client reports to the health care providers office stating that her lochia has changed from lochia alba to lochia rubra. Which does the nurse suspect?

Retained placental fragments Explanation: The nurse suspects a late postpartum hemorrhage since the lochia has progressed to lochia alba and then it returned to lochia rubria. Late postpartum hemorrhage is typically due to subinvolution secondary to retained placental fragments, distended bladder, uterine myoma, and infection. Uterine atony, lacerations, and uterine inversion would most likely lead to early postpartum hemorrhage.

When making a home visit, the nurse observes a newborn sleeping on his back in a bassinet. In one corner of the bassinet is some soft bedding material, and at the other end is a bulb syringe. The nurse determines that the mother needs additional teaching for which reason?

Soft bedding material should not be in areas where infants sleep. The nurse should instruct the mother to remove all fluffy bedding, quilts, stuffed animals, and pillows from the crib to prevent suffocation. Newborns and infants should be placed on their backs to sleep. Having the bulb syringe nearby in the bassinet is appropriate. Although a crib is the safest sleeping location, a bassinet is appropriate initially.

Which vaccine is safe to give in pregnancy?

The flu vaccine after the first trimester Live viruses vaccines such as MMR, varicella, and HPV are contraindicated in pregnancy.

The nurse is preparing to assess the duration of contractions for a patient in labor. Which process should the nurse use to time the contractions?

The interval between the beginning and the end of one contraction Explanation: To determine the beginning of a contraction without a monitor, rest a hand on a woman's abdomen at the fundus of the uterus very gently until you sense the gradual tensing and upward rising of the fundus that accompanies a contraction. Time the duration of the contraction from the moment the uterus first tenses until it has relaxed again. This is the duration. Contractions are not timed by measuring the number of contractions in 5 minutes, the end of one contraction to the beginning of the next, or by using the interval between the acmes of two consecutive contractions.

The nurse is examining a newborn and notes that there is bruising and swelling on the newborn's head, limited to the right side of the scalp and does not cross the suture lines. How will the nurse interpret this finding?

This is a cephalohematoma that typically spontaneously resolves without interventions. Explanation: The newborn is exhibiting signs of a cephalohematoma, a collection of blood under the periosteum of the skull. It is caused by birth trauma and should resolve spontaneously. If the swelling crosses the suture line, caput succedaneum is suspected. A subarachnoid hemorrhage in a newborn usually results in symptoms such as seizures, apnea, and bradycardia.

The laboring client who is at 3 cm dilation (dilatation) and 25% effaced is asking for analgesia. The nurse explains the analgesia usually is not administered prior to the establishment of the active phase. What is the appropriate rationale for this practice?

This may prolong labor and increase complications. Explanation: Administration of pharmacologic agents too early in labor can stall the labor and lengthen the entire labor. The client should be offered nonpharmacologic options at this point until she is in active labor.

A new mother who is breastfeeding her newborn asks the nurse, "How will I know if my baby is drinking enough?" Which response by the nurse would be most appropriate?

"He should wet between 6 to 10 diapers each day." Explanation: Soaking 6 to 10diapers a day indicates adequate hydration. Contentedness after feeding is not an indicator for adequate hydration. Typically a newborn wakes up 8 to 12 times per day for feeding. As the infant gets older, the time on the breast increases. Moist mucous membranes help to suggest adequate hydration, but this is not the best indicator.

A patient who gave birth 2 hours ago expresses concern about her baby developing jaundice. How should the nurse respond? Choose the best response.

"I understand your concern because as many as 50% of babies can develop jaundice." Explanation: As many as 50% of term newborns will develop physiologic jaundice. Physiologic jaundice occurs after the first 24 hours of life and is not pathologic. Pathologic jaundice will develop within the first 24 hours of life.

A new mother is concerned because it is 24 hours after birth and her breasts have still not become engorged with breast milk. How should the nurse respond to this concern?

"It takes about 3 days after birth for milk to begin forming." The formation of breast milk (lactation) begins in a postpartal woman regardless of her plans for feeding. For the first 2 days after birth, an average woman notices little change in her breasts from the way they were during pregnancy, since midway through pregnancy she has been secreting colostrum, a thin, watery, prelactation secretion. On the third day postpartum, her breasts become full and feel tense or tender as milk forms within breast ducts and replaces colostrum. There is no need to recommend formula feeding to the mother. Mastitis is inflammation of the lactiferous (milk-producing) glands of the breast; there is no indication that the client has this condition. Lactational amenorrhea is the absence of menstrual flow that occurs in many women during the lactation period.

The nurse is caring for a couple who are trying to get pregnant and have not been able to for over a year. The nurse explains to the couple that diagnostic testing usually begins on the male partner, as these tests are easier. The couple asks what kind of problems a man can have that can cause infertility. What should be the nurse's response?

"Men can have problems that increase the temperature around their testicles, which decreases the quality of their semen." Explanation: Men may be affected by varicoceles, varicose veins around the testicle, which decrease semen quality by increasing testicular temperature. Retrograde ejaculation or ejaculation into the bladder is assessed by urinalysis after ejaculation. Blood tests for male partners may include measuring testosterone; FSH and LH (both of which are involved in maintaining testicular function); and prolactin levels. Therefore options A, C, and D are incorrect.

The nurse realizes the educational session conducted on due dates was successful when a participant is overheard making which statement?

"The ability of my placenta to provide adequate oxygen and nutrients to my baby after 42 weeks' gestation is thought to be compromised." Explanation: The ability of the placenta to provide adequate oxygen and nutrients to the fetus after 42 weeks' gestation is thought to be compromised, leading to perinatal mortality and morbidity. After 42 weeks the placenta begins aging. Deposits of fibrin and calcium, along with hemorrhagic infarcts, occur and the placental blood vessels begin to degenerate. All of these changes affect diffusion of oxygen to the fetus. As the placenta loses its ability to nourish the fetus, the fetus uses stored nutrients to stay alive, and wasting occurs.

A woman's baby is HIV positive at birth. She asks the nurse if this means the baby will develop AIDS. Which statement would be the nurse's best answer?

"The antibodies may be those transferred across the placenta; the baby may not develop AIDS." Explanation: Infants born of HIV-positive women test positive for HIV antibodies at birth because these have crossed the placenta. An accurate disease status cannot be determined until the antibodies fade at about 18 months. Testing positive for HIV antibodies does not mean the infant has AIDS. Having a cesarean birth does decrease the risk of transmitting the virus to the infant at birth; it does not prevent the transmission of the disease. HIV antibodies do cross the placenta, which is why babies born of HIV positive mothers are HIV positive.

The pregnant woman with diabetes asks the nurse why her last baby weighed 11 pounds. What is the best response by the nurse?

"The fetus increases insulin production in response to elevated glucose levels of the mother, which acts as a fetal growth hormone." Explanation: Consistently elevated fetal insulin levels cause the distinctive growth pattern. Because maternal glucose levels are elevated and glucose readily crosses the placenta, the fetus responds by increasing insulin production. Because insulin acts as a fetal growth hormone, consistently high levels cause fetal macrosomia, birth weight of greater than 4,500 g. Insulin also causes disproportionate fat buildup to the shoulders and upper body, increasing the risk for shoulder dystocia and birth trauma.

A couple has just been notified that their unborn child carries a genetic disorder. The couple expresses concern that the insurance company will not cover the costs associated with the medical bills for the child. What is the most appropriate response by the nurse?

"There are laws in place that prohibit that from happening." The Genetic Information Nondiscrimination Act of 2008 prohibits insurance companies from denying coverage or charging higher premiums based solely on genetic predisposition.

A client in her second trimester of pregnancy is anxious about the blotchy, brown pigmentation appearing on her forehead and cheeks. She also reports increased pigmentation on her breasts and genitalia. Which statement by the nurse is most appropriate?

"This is called facial melasma and should fade after the birth." Explanation: The skin and complexion of pregnant women undergo hyperpigmentation, primarily as a result of estrogen, progesterone, and melanocyte stimulating hormone levels. The increased pigmentation that occurs on the breasts and genitalia also develops on the face to form the "mask of pregnancy," or facial melasma (chloasma). This is a blotchy, brownish pigment that covers the forehead and cheeks in dark-haired women. The nurse would inform the client that this is a normal occurrence in pregnancy and should fade after birth.

When providing preconception care to a client, which instruction would be most important for the nurse to provide about medications during pregnancy?

"You need to talk with your provider about using medications." Medication use is common during pregnancy, with prevalence estimates generally exceeding 65% and increasing over the years. Pregnant women use a wide variety of both prescription and OTC medications for both pregnancy-related conditions and conditions unrelated to pregnancy conditions (van Gelder, de Jong, Winkel et al., 2019). Little is known about the effects of taking most medications during pregnancy. It is best for pregnant women to not take any medications during their pregnancy. At the very least, they should be encouraged to discuss with the health care provider their current medications and any herbal remedies they take so that they can learn about any potential risks should they continue to take them during pregnancy. A common concern of many pregnant women involves the use of OTC medications and herbal agents. Many women consider these products benign simply because they are available without a prescription (King et al., 2019). Although herbal medications are commonly thought of as "natural" alternatives to other medicines, they can be just as potent as some prescription medications. The nurse should encourage pregnant women to check with their health care providers before taking anything.

With the administration of oxygen, a preterm infant's Pa02 level is monitored carefully. It is important to keep this level under which value to help prevent retinopathy of prematurity?

100 mm Hg Explanation: Retinal capillaries can be damaged by excessive oxygen levels. Keeping the Pa02 level under 100 mm Hg helps prevent this.

A low-risk client is in the active phase of labor. The nurse evaluates the fetal monitor strip at 10:00 a.m. and notes the following: moderate variability, FHR in the 130s, occasional accelerations, and no decelerations. At what time should the nurse reevaluate the FHR?

10:30 a.m. Explanation: Assess and document fetal status at least every 30 minutes. Record the baseline FHR every 30 minutes and evaluate the fetal monitor tracing for abnormal patterns. Variability should be present, except for brief periods of fetal sleep or when the mother receives narcotics or other selected medications, and no late decelerations should be present. Accelerations of the FHR are normal.

The parents are questioning why their newborn was born deaf when there are no other deaf family members. The nurse could explore possible exposure to a teratogenic agent at which stage of the pregnancy?

6 weeks Explanation: Teratogenic agents ingested during the embryonic stage (2 to 8 weeks) can affect the neurologic system of the fetus, including the hearing. During weeks 6 through 8, the ear is most vulnerable to teratogenic agents. From the preembryonic stage of fertilization to the beginning of week 2, there is decreased risk due to no implantation or transfer of substances from the mother to the developing blastocyte. During the fetal stage (9 weeks to birth), the fetus is fully formed and is now concentrating on increasing in size. There is a decreased risk from teratogenics during this time period.

A nurse is performing a physical examination of a pregnant woman. During examination, the nurse notices a bruise on the client's abdomen. Which action should the nurse take?

Ask the client to account for the bruise. Explanation: Many women seen in emergency departments are there because they have been maltreated by their intimate partner. Common injuries suffered include burns, lacerations, bruises, and head injuries. Asking all women at physical examinations to account for any bruise they have helps detect physical maltreatment. The nurse should not assume that the bruise is just the result of an accident or that it is the result of intimate partner violence; she should question the client to learn more. Depending on how the client responds, it may be appropriate to make a note in the client's chart or to take some other action to help protect the client.

Which assessment by the nurse will best monitor the nutrition and fluid balance in the postterm newborn?

Assess for decrease in urinary output. The nurse should assess for a decrease in urinary output and fluid balance in the preterm or postterm newborn. Weight of the newborn should be measured daily, not once every 2 to 3 days. Increased muscle tone does not indicate nutrition and fluid imbalance. A rise, not fall, in temperature indicates dehydration.

A patient with asthma who is 32 weeks' pregnant is concerned that the health care provider has reduced the doses of asthma maintenance medications. How should the nurse respond to this patient's concern?

Asthma medication may reduce labor contractions and should be reduced. Explanation: Some asthma maintenance medication such as beta-adrenergic agonists may be taken safely during pregnancy, but they have the potential to reduce labor contractions. The doses of these medications may be reduced as the patient approaches the time of delivery. Not all asthma medication is teratogenic. Asthma can improve during pregnancy because of circulating corticosteroids; however, the doses of the medications should have already been adjusted according to the patient's symptoms. There is no evidence to support that asthma medication is ineffective during pregnancy.

A postpartum patient is prescribed docusate sodium (Colace) as treatment for constipation. What should the nurse include when teaching the patient about this medication?

Be sure to engage in activity to aid in intestinal motility. This medication works the best when a high-fiber diet is consumed. Take each dose of the medication with a full glass of water or juice. Docusate sodium (Colace) is used in the postpartal period to prevent constipation. It works by lowering the surface tension of feces, allowing water and lipids to penetrate the stool and soften it. The nurse should instruct the patient to engage in activity to promote intestinal motility, consume a diet high in fiber, and take each dose of the medication with a full glass of water or juice. This medication has abdominal pain and diarrhea as potential adverse effects. This medication is not taken after every meal but rather one dose per day.

The nurse in a busy L & D unit is caring for a woman beginning induction via oxytocin drip. Which prescription should the nurse question with regard to titrating the infusion upward for adequate contractions?

Begin infusion at 10 milliunits (mu)/min and titrate every 15 minutes upward by 5 mu/min. Explanation: Hyperstimulation is usually defined as five or more contractions in a 10-minute period or contractions lasting more than 2 minutes in duration or occurring within 60 seconds of each other. The surest method to relieve hyperstimulation is to immediately discontinue the oxytocin infusion. The rate should not be increased by more than 2 milliunits at a time. When the infusion is administered, the oxytocin solution should be "piggybacked" to a maintenance IV solution such as Ringer's lactate and the piggyback added to the main infusion at the port closest to the woman. Infusions are usually begun at a rate of 1 to 2 milliunits/min. If there is no response, the infusion is gradually increased every 30 to 60 minutes by small increments of 1 to 2 milliunits/min until contractions begin.

The nurse is comforting and listening to a young couple who just suffered a spontaneous abortion (miscarriage). When asked why this happened, which reason should the nurse share as a common cause?

Chromosomal abnormality Explanation: The most common cause for the loss of a fetus in the first trimester is associated with a genetic defect or chromosomal abnormality. There is nothing that can be done and the mother should feel no fault. The nurse needs to encourage the parents to speak with a health care provider for further information and questions related to genetic testing. Early pregnancy loss is not associated with maternal smoking, lack of prenatal care, or the age of the mother.

A nurse is providing care to a newborn and places a warm towel on a cold scale to prevent heat loss by which mechanism?

Conduction Explanation: Conduction involves the loss of heat through direct contact with an object that is cooler. Placing a warm towel on a cold scale reduces heat loss by this mechanism. Evaporation involves the loss of heat through conversion of water to its gaseous state. Drying the newborn quickly after bathing reduces heat loss by this mechanism. Convection involves the loss of heat through exchange between two objects within the same environment. Avoiding drafts and using an isolette help reduce heat loss by this mechanism. Radiation involves the loss of head across a gradient between two objects that are not in direct contact with one another. Not placing a bassinet near cold walls or windows helps reduce heat loss by this mechanism.

A breastfeeding client presents with a temperature of 102.4°F (39°C) and a pulse of 110 bpm. She reports general fatigue and achy joints, and her left breast is engorged, red, and tender. Which instructions would the nurse anticipate being given to this client? Select all that apply.

Continue breastfeeding on the left side, if the infant is willing to latch on. Take prescribed antibiotics until all prescribed doses are completed. If infant refuses to feed, pump the breast to maintain flow. An infection of the breast during lactation is termed mastitis. Mastitis can interfere with lactation, and sometimes an infant will refuse to nurse on the affected side. The women's medical provider must be notified to initiate antibiotic treatment. Mothers should be instructed to continue breastfeeding if the infant will breastfeed from the affected side. If the infant refuses, instruct the mother to pump her breasts to maintain flow (and to avoid clogged ducts) and then offer the affected breast after 12 to 24 hours. Unless specifically directed otherwise, infants are safe to continue to breastfeed while a mother is being treated for mastitis; there is no reason to provide alternative feeding methods or to wean because of maternal mastitis.

A woman gave birth vaginally approximately 12 hours ago, and her temperature is now 100° F (37.8° C). Which action would be most appropriate?

Continue to monitor the woman's temperature every 4 hours; this finding is normal. A temperature of 100.4° F (38° C) or less during the first 24 hours postpartum is normal and may be the result of dehydration due to fluid loss during labor. There is no need to notify the health care provider, obtain a urine culture, or inspect the perineum (other than the routine assessment of the perineum) because this finding is normal.

Which change best describes the insulin needs of a client with type 1 diabetes mellitus who has just delivered an infant vaginally without complications?

Decreased Explanation: The placenta produces the hormone human placental lactogen, an insulin antagonist. After birth, the placenta, the major source of insulin resistance, is gone. Insulin needs decrease, and women with type 1 diabetes may need only one-half to two-thirds of the prenatal insulin dose during the first few postpartum days. Blood glucose levels should be monitored and insulin dosages adjusted as needed. The client should be encouraged to maintain appropriate dietary schedules, even if her infant is feeding on demand.

Which response is most appropriate for a client with diabetes who wants to breast-feed but is concerned about the effects of breastfeeding on her health?

Diabetic clients may breast-feed; insulin requirements may decrease from breast-feeding Explanation: Breast-feeding has an antidiabetic effect. Insulin needs are decreased because carbohydrates are used in milk production. Breastfeeding clients are at a higher risk for hypoglycemia in the first postpartum days after birth because glucose levels are lower. Diabetic clients should be encouraged to breast-feed.

A pregnant patient is prescribed to have labor induced with oxytocin. How should the nurse prepare to administer this medication?

Diluted as a "piggyback" infusion Explanation: When administering oxytocin, the infusion should be "piggybacked" to a maintenance IV solution and add the piggyback to the main infusion at the port closest to the patient. If the oxytocin needs to be discontinued quickly during the induction, little solution remains in the tubing to still infuse, and the main IV line can still be maintained. Oxytocin is not administered as an intravenous bolus, as intramuscular injections, nor is it diluted in the main intravenous fluid.

After a rapid assessment determines that a newborn is in need of resuscitation, the nurse would perform which action first?

Dry the newborn thoroughly. Explanation: If resuscitation is needed, the nurse must first stabilize the newborn by drying the newborn thoroughly with a warm towel and provide warmth by placing him or her under a radiant heater to prevent rapid heat loss. Next the newborn's head is placed in a neutral position to open the airway, and the airway is cleared with a bulb syringe or suction catheter. Breathing is stimulated. Often handling and rubbing the newborn with a dry towel may be all that is needed to stimulate respirations. Next ventilations and then chest compressions are done. Administration of epinephrine and/or volume expanders is the last step.

Which assessment findings are most prominent in the infant with Tetralogy of Fallot and significant pulmonary stenosis?

Dyspnea on limited exertion, fatigue, cyanosis Explanation: The infant with Tetralogy of Fallot and significant pulmonary stenosis exhibits prominent signs of dyspnea, fatigue and cyanosis. Other symptoms include feeding difficulties and poor weight gain, retarded growth and development and breathlessness. Irregular heartrate, dry mucous membranes, nausea and decreased muscle tone may be present in some form but are not the prominent signs.

The labor and delivery nurse would identify that which women are more likely to experience dystocia during labor?

Dystocia is any labor with an abnormally slow or fast progression. Causes of dystocia are diverse. Factors associated with dystocia include Asian, African, or Hispanic descent; diabetes; oxytocin augmentation; history of operative delivery with previous pregnancy; and increased fetal weight.

What are small unopened or plugged sebaceous glands that occur in a newborn's mouth and gums?

Epstein pearls Explanation: Unopened sebaceous glands are generally called milia. When they are in the mouth and gums, they are called Epstein pearls.

The nurse is caring for a newborn who was delivered via a planned cesarean delivery. The nurse determines the infant requires closer monitoring than a vaginal delivery infant based on which factor?

Excessive fluid in its lungs, making respiratory adaptation more challenging. Explanation: During a vaginal birth the infant is squeezed by uterine contractions, which squeeze fluid out of the lungs and prepare them for breathing. The infant who is born via cesarean delivery without labor first does not have the mechanical removal of the fluid from the lungs. This places the infant at increased risk for respiratory compromise, so there is a need to more closely assess a newborn after birth. The lungs should inflate once the baby is delivered and not wait until the amniotic fluid is absorbed. The umbilical cord is not clamped until the infant is out of the womb and starts to take its first breaths.

In which manner is the fetal status best assessed during the active and transition stages of labor?

Fetal heart rate at the peak of a contraction Explanation: Normal labor stresses the fetus by increasing intracranial pressure, decreasing heart rate and placental blood flow. Assessing the fetal heart rate during the peak of the contraction best indicates how the fetus is tolerating the labor. Fetal movement and fetal kicks can indicate that a fetus is compromised but that is a late sign of distress.

A pregnant client with deep vein thrombosis has been diagnosed as having systemic lupus erythematosus (SLE). The nurse would monitor the client closely for the development of which complication?

Fetal malnutrition Explanation: SLE is an autoimmune disorder in which there is deposition of immune complexes in the capillaries and visceral structures. Clients with SLE who become pregnant are at an increased risk of fetal malnutrition due to decreased placental circulation. Pregnancy-related problems in SLE include prematurity, stillbirth, decreased placental weight and thinner placental villi. In clients with SLE there is preterm birth and decreased placental weight. Fetal macrosomia is seen in clients having gestational diabetes, not SLE.

what are fetal risks associated with post-term pregnancies?

Fetal risks associated with a post-term pregnancy include macrosomia, shoulder dystocia, brachial plexus injuries, low Apgar scores, postmaturity syndrome (loss of subcutaneous fat and muscle and meconium staining), and cephalopelvic disproportion. As the placenta ages, its perfusion decreases and it becomes less efficient at delivering oxygen and nutrients to the fetus. Amniotic fluid volume also begins to decline after 38 weeks' gestation, possibly leading to oligohydramnios, subsequently resulting in fetal hypoxia and an increased risk of cord compression because the cushioning effect offered by adequate fluid is no longer present. Hypoxia and oligohydramnios predispose the fetus to aspiration of meconium, which is released by the fetus in response to a hypoxic insult (Norwitz, 2019). All of these issues can compromise fetal well-being and lead to fetal distress.

The nurse assesses an infant's body temperature as 97.1°F (36.2°C) during an extended resuscitation at birth. What consequence of this temperature would the nurse anticipate?

Fetal shunts remain open. Anaerobic glycolysis occurs. Pulmonary perfusion decreases. Metabolism increases. Explanation: When the infant's body temperature is low, the fetal shunts remain open, anaerobic glycolysis occurs, pulmonary perfusion decreases, and metabolism increases. Immune function is not a consequence of body temperature.

The nurse is assessing the fundus of a patient on postpartum day 2. What should the nurse expect when palpating the fundus?

Fundus two fingerbreadths below umbilicus and firm Because uterine contraction begins immediately after placental delivery, the fundus of the uterus is palpable through the abdominal wall, halfway between the umbilicus and the symphysis pubis, within a few minutes after birth. One hour later, it will rise to the level of the umbilicus, where it remains for approximately the next 24 hours. From then on, it decreases one fingerbreadth or centimeter per day and will be palpable 1 cm below the umbilicus. For the second postpartal day, the uterus will be two fingerbreadths or centimeters below the umbilicus. The fundus should not be palpated 4 cm above the symphysis pubis, 4 cm below the umbilicus, or two fingerbreadths above the symphysis pubis on the second postpartum day. The fundus should not be hard.

A multigravida client is pregnant for the third time. Her previous two pregnancies ended in an abortion in the first and third month of pregnancy. How will the nurse classify her pregnancy history?

G3 P0020 Gravida (G) is the total number of pregnancies she has had, including the present one. Therefore she is G3 and not G2. Para (P), the outcome of her pregnancies, is further classified by the FPAL system as follows: F = Full term: number of babies born at 37 or more weeks of gestation, which is 0 and not 1 in this case. P = Preterm: number of babies born between 20 and 37 weeks of gestation, which is 0 in this case. A = Abortions: total number of spontaneous and elective abortions, which is 2 in this case. L = Living children, as of today. She has no living children; therefore, it is 0 and not 1.

A pregnant client arrives for her first prenatal appointment. She reports her previous pregnancy ended at 19 weeks, and she has 3-year-old twins born at 30 weeks' gestation. How will the nurse document this in her records?

G3 T0 P1 A1 L2 G indicates the total number of pregnancies (2 prior, now pregnant = 3); T indicates term deliveries at or beyond 38 weeks' gestation (none = 0); P is for preterm deliveries (at 20 to 37 weeks = 1; multiple fetus delivery are scored as 1); A is for abortions or pregnancies ending before 20 weeks' gestation (1); and L refers to living children which is 2. Thus, G3 T0 P1 A1 L2 is what the nurse should note in the client's record.

The obstetrician is examining a woman who is in early labor to determine the positioning of the fetus. The nurse knows that which of the following fetal attitudes would be the most advantageous for birth?

Head flexed forward so much that the chin touches the sternum Explanation: A fetus in good attitude is in complete flexion: the spinal column is bowed forward, the head is flexed forward so much that the chin touches the sternum, the arms are flexed and folded on the chest, the thighs are flexed onto the abdomen, and the calves are pressed against the posterior aspect of the thighs. This usual "fetal position" is advantageous for birth because it helps a fetus present the smallest anteroposterior diameter of the skull to the pelvis and also because it puts the whole body into an ovoid shape, occupying the smallest space possible. The other attitudes listed are not ideal, because larger diameters of the fetus' skull are presenting.

The nurse is monitoring the EFM and notes the following: variable V-shaped decelerations in the FHR lasting about 30 seconds, accelerations of about 5 bpm before and after each deceleration, no overshoot, and baseline FHR within normal limits. Which response should the nurse prioritize?

Help the woman change positions. Explanation: Changing to a different position is a first intervention to determine if this will improve the oxygen to the fetus. It may not necessarily mean to elevate the left hip with a pillow. The client could try sitting up and dangling her feet. Supplemental oxygen should be maintained until the mother is stable. Placing the client on her side may increase the work of breathing. Pharmacologic interventions are premature. Lying on the side with the left hip on a pillow is often used to correct postural hypotension related to the vena cava being compressed by the pregnant uterus.

The nurse is caring for a diabetic, postpartum client who has developed an infection.The nurse is aware that infections in diabetic clients tend to be more severe and can quickly lead to complications. Which complication should the nurse assess this client for?

Ketoacidosis Explanation: Diabetic clients who become pregnant tend to become sicker and develop illnesses more quickly than pregnant clients without diabetes. Severe infections in diabetes can lead to diabetic ketoacidosis (body produces excess blood acids; less insulin in body casueses harmful ketoacids; can also be caused by infection). Anemia, respiratory acidosis, and respiratory alkalosis aren't generally associated with infections in diabetic clients.

The nurse is caring for a patient in labor whose fetus is in an occiput posterior position. Which intervention should the nurse use to reduce this patient's discomfort?

Massage the lower back. Explanation: Because the fetal head rotates against the sacrum in the occiput posterior position, the patient may experience pressure and pain in the lower back because of sacral nerve compression. Applying counter pressure on the sacrum by a back rub may be helpful in relieving a portion of the pain. The patient does not need to be placed in the prone or Trendelenburg positions. Ice packs are not indicated to reduce this pain.

A postpartum patient is prescribed methylergonovine 0.2 mg for uterine subinvolution. Which action should the nurse take before administering the medication to the patient?

Measure blood pressure. Explanation: Methylergonovine can increase blood pressure and must be used with caution in patients with hypertension. The nurse should assess the blood pressure prior to administrating and about 15 minutes afterward to detect this side effect. Methylergonovine does not affect ambulation, urine output, or hematocrit level.

Which recommendation should be given to a client with mastitis who is concerned about breastfeeding her neonate?

She should continue to breastfeed; mastitis will not infect the neonate. Explanation: The client with mastitis should be encouraged to continue breastfeeding while taking antibiotics for the infection. No supplemental feedings are necessary because breastfeeding does not need to be altered and actually encourages resolution of the infection. Analgesics are safe and should be administered as needed.

Which of the following is TRUE regarding intimate partner violence?

One in four women will be a victim of violence. Explanation: Approximately 20% to 30% of women will be a victim of violence. Women experience an increase in violence during pregnancy. Abusers often have no outward signs that they are abusers or have a mental illness. Women usually blame themselves and feel they cannot leave the relationship.

A 17-year-old primigravida with type 1 diabetes is at 37 weeks' gestation comes to the clinic for an evaluation. The nurse notes her blood sugar has been poorly controlled and the health care provider is suspecting the fetus has macrosomia. The nurse predicts which step will be completed next?

Preparing for amniocentesis and fetal lung maturity assessment Explanation: If the infant has macrosomia (large for gestational age weighs more than 8 lbs 13 ounces or 4000 g), is large for gestation age, and the mother has had poor blood-sugar control, the provider will want further information on the fetus and readiness for delivery before making any decisions on delivery. After determining the readiness of the fetus, then plans for delivery can be determined and scheduled.

The nurse is assisting a health care provider in inserting an epidural into a laboring mother. Completion of which nursing task helps prevent maternal hypotension?

Priming tubing for initiating a fluid bolus Explanation: Priming tubing for a fluid bolus is helpful in preventing maternal hypotension secondary to epidural placement. Introducing fluid to the vascular space elevates the circulating volume and blood pressure. Patterned breathing helps to promote relaxation but does not influence hypotension. Elevating the client's legs assists in returning the blood to the heart. This may be helpful but not as helpful as the fluid bolus. Vasopressors are not administered during labor. Fluid is completed before medication would be introduced into the system.

Blood work results show that a client has a luteal defect, which is suspected to be the primary cause of her subfertility. The nurse explains that this condition is generally corrected by which of the following?

Progesterone vaginal suppositories Explanation: If the problem of subfertility appears to be a luteal phase defect, this can be corrected by progesterone vaginal suppositories begun on the third day of a woman's temperature rise and continued for the next 6 weeks (if pregnancy occurs) or until a menstrual flow begins. If a myoma (fibroid tumor) or intrauterine adhesions are found to be interfering with fertility, a myomectomy, or surgical removal of the tumor and adhesions, can be scheduled. If the subfertility problem is identified as tubal insufficiency from inflammation, the prescription of diathermy or steroid administration may be helpful to reduce adhesions.

The nurse is concerned that a pregnant patient is experiencing abruptio placentae. What did the nurse assess in this patient?

Sharp fundal pain and discomfort between contractions Explanation: Abruptio placentae is characterized by a sharp, stabbing pain high in the uterine fundus as the initial separation occurs. Manifestations of abruptio placentae do not include increased blood pressure, oliguria, pain in the lower quadrant, increased pule rate, painless vaginal bleeding, or a fall in blood pressure.

A woman who has sickle cell anemia asks the nurse if her infant will develop sickle cell disease. The nurse would base the answer on which information?

Sickle cell anemia is recessively inherited. Explanation: Sickle cell anemia is an autosomal recessive disease requiring that the person have two genes for the disease, one from each parent. If one parent has the disease and the other is free of the disease and trait, the chance of the child inheriting the disease is zero. If the woman has the disease and her partner has the trait, there is a 50% chance that the child will be born with the disease. If both parents have the disease, then all of their children also will have the disease.

When providing care for a newborn to a mother who was a smoker during her pregnancy, the nurse will anticipate the size of the infant to be what?

Smaller than average Explanation: Smoking decreases the blood flow to the fetus. This is one cause for the infant to be smaller than average at birth. Mothers who smoke do not generally have infants who are larger than average; diabetic mothers generally have infants who are larger than average. An average size infant is not generally born to a mother who smokes.

A female patient had a diagnostic laparoscopy and is now reporting a sharp pain in her shoulder. What response is a priority by the nurse?

Some of the carbon dioxide that was used to insufflate the abdomen has likely escaped under the diaphragm and this will resolve on its own. Women may feel bloating of the abdomen from the infusion of CO2 after such a procedure. If some CO2 escapes under the diaphragm, there may be extremely sharp should pain from pressure.

Over the past 20 weeks, the following blood pressure readings are documented for a pregnant client with chronic hypertension: week 16 - 130/86 mm Hg; week 20 - 138/88 mm Hg; week 24 - 136/82 mm Hg; and week 28 - 138/88 mm Hg. The nurse interprets these findings as indicating which classification of her blood pressure?

Stage 1 Explanation: Chronic hypertension exists when the woman has high blood pressure before pregnancy or before the 20th week of gestation, or when hypertension persists for more than 12 weeks. The Eighth Report of the Joint National Committee on Prevention, Detection, Evaluation, and Treatment of High Blood Pressure (Joint National Committee [JNC 8], 2018) blood pressure guidelines classify hypertension as follows: elevated: Systolic between 120 and 129 mm Hg and diastolic less than 80 mm Hg; Stage 1: Systolic between 130 and 139 mm Hg or diastolic between 80-89 mm Hg; Stage 2:Systolic at least 140 mm Hg or diastolic at least 90 mm Hg; Hypertensive crisis: Systolic over 180 mm Hg and/or diastolic over 120 mm Hg (Alexander, 2019; Bakris, 2019). The client has stage 1 hypertension.

A nurse is caring for a postpartum client who has been treated for deep vein thrombosis (DVT). Which prescription would the nurse question?

Take an oral contraceptive daily. Explanation: When caring for a client with DVT, the nurse should instruct the client to avoid using oral contraceptives. Cigarette smoking, use of oral contraceptives, sedentary lifestyle, and obesity increase the risk for developing DVT. The nurse should encourage the client with DVT to wear compression stockings. The nurse should instruct the client to avoid using products containing aspirin when caring for clients with bleeding, but not for clients with DVT. Prolonged rest periods should be avoided. Prolonged rest involves staying motionless; this could lead to venous stasis, which needs to be avoided in cases of DVT.

The nurse is explaining the discharge instructions to a client who has developed postpartum cystitis. The client indicates she is not drinking a glass of fluid every hour because it hurts too much when she urinates. What is the best response from the nurse?

Teach that adequate hydration helps clear the infection quicker. Explanation: Adequate hydration is necessary to dilute the bacterial concentration in the urine and aid in clearing the organisms from the urinary tract. Encourage the woman to drink at least 3000 mL of fluid a day, suggesting she drink one glass per hour. Drinking fluid will make the urine acidic, deterring organism growth. The other choices are also options but address the symptoms and not the root cause. The goal should be to rid the body of the infection, not concentrate on counteracting the results of the infection.

The nurse has administered erythromycin ointment to a newborn. What outcome indicates this nursing intervention has been effective?

The newborn does not contract ophthalmia neonatorum. Explanation: Eye prophylaxis is given to prevent (not treat) ophthalmia neonatorum, a severe eye infection contracted in the birth canal of a woman with gonorrhea or chlamydia. This is unrelated to tear production or jaundice.

The nurse is caring for a client who has an irregular pattern of uterine contraction. As a result, the nurse anticipates a problem with which?

The powers Explanation: One of the four "P's" is the power of the uterine contractions. This power begins with regular contractions which become closer together and increase in intensity. The powers push the fetus down the birth canal.

A group of nursing students are preparing a presentation depicting the fetal circulation. The instructor determines the presentation is successful when the students correctly illustrate which route for the ductus arteriosus?

The pulmonary artery to the aorta Explanation: Because the fetal lungs are not inflated, blood must be diverted past them. The ductus arteriosus helps to do this by shunting blood from the pulmonary artery to the aorta. The foramen ovale diverts blood from the right atrium to the left atrium, bypassing the lungs. The ductus venosus diverts a portion of blood from the umbilicus to the inferior vena cava instead of passing through the liver first.

A client experiencing contractions presents at a health care facility. Assessment conducted by the nurse reveals that the client has been experiencing Braxton Hicks contractions. The nurse has to educate the client on the usefulness of Braxton Hicks contractions. Which role do Braxton Hicks contractions play in aiding labor

These contractions help in softening and ripening the cervix. Explanation: Braxton Hicks contractions assist in labor by ripening and softening the cervix and moving the cervix from a posterior position to an anterior position. Prostaglandin levels increase late in pregnancy secondary to elevated estrogen levels; this is not due to the occurrence of Braxton Hicks contractions. Braxton Hicks contractions do not help in bringing about oxytocin sensitivity. Occurrence of lightening, not Braxton Hicks contractions, makes maternal breathing easier.

A woman is only 30 weeks pregnant, but the physician determines that the fetus must be delivered for the safety of the mother. The physician orders a glucocorticosteroid injection to be given. Why does the physician order this injection?

To promote the formation of surfactant in the fetal lungs Explanation: A key task of the third trimester is maturation of the fetal lungs. Growth factors in amniotic fluid promote growth and differentiation of lung tissue. With normal amniotic fluid volume, functionality of the lungs depends on their ability to form surfactant, which prevents collapse of the alveoli upon expiration. If a fetus must be delivered between 28 and 34 weeks, a glucocorticosteroid injection is given to the mother to promote the formation of surfactant.

Immediately following an epidural block, a pregnant patient's blood pressure suddenly falls to 90/50 mmHg. What action should the nurse take first?

Turn onto the left side or raise the legs. Explanation: To help prevent supine hypotension syndrome, place the pregnant patient on the left side after an epidural block. If hypotension should occur, the patient's legs should be raised in addition to providing oxygen, intravenous fluids, and medication. The supine position encourages hypotension syndrome. Raising the head of the bed and deep breathing are not interventions to help with hypotension syndrome.

A nurse caring for a pregnant client in labor observes that the fetal heart rate (FHR) is below 110 beats per minute. Which interventions should the nurse perform? Select all that apply.

Turn the client on her left side. Administer oxygen by mask. Assess client for underlying causes. Explanation: The nurse should turn the client on her left side to increase placental perfusion, administer oxygen by mask to increase fetal oxygenation, and assess the client for any underlying contributing causes. The client's questions should not be ignored; instead, the client should be reassured that interventions are to effect FHR pattern change. A reduced IV rate would decrease intravascular volume, affecting the FHR further.

A nurse is caring for a client who was raped at gunpoint. The client does not want any photos taken of the injuries. The client also does not want the police to be informed about the incident even though state laws require reporting life-threatening injuries. Which intervention should the nurse perform to document and report the findings of the case?

Use direct quotes and specific language. Explanation: The nurse should use direct quotes and specific language as much as possible when documenting. The nurse should not obtain photos of the client without informed consent. The nurse should, however, document the refusal of the client to be photographed. Documentation must include details as to the frequency and severity of abuse and the location, extent, and outcome of injuries, not just a description of the interventions taken. The nurse is required by law to inform the police of any injuries that involve knives, firearms, or other deadly weapons or that present life-threatening injuries.

A nurse is caring for a non-breastfeeding client in the postpartum period. The client reports engorgement. What suggestion should the nurse provide to alleviate breast discomfort?

Wear a well-fitting bra. The nurse should suggest the client wear a well-fitting bra to provide support and help alleviate breast discomfort. Application of warm compresses and expressing milk frequently are suggested to alleviate breast engorgement in breastfeeding clients. Hydrogel dressings are used prophylactically in treating nipple pain.

The nurse places a newborn experiencing respiratory difficulty under a radiant warmer to prevent which complication?

acidosis Explanation: A radiant warmer is use to keep the infant warm. When an infant is cold, brown fat metabolism leads to acidosis, which would further complicate respiratory difficulties.

The nurse is concerned that a client is not obtaining enough folic acid. Which test would the nurse anticipate being used to evaluate the fetus for potential neural tube defects?

alpha-fetoprotein analysis Explanation: Alpha-fetoprotein is a substance produced by the fetus. AFP enters the maternal circulation by crossing the placenta. If there is a developmental defect, more AFP escapes into amniotic fluid from the fetus. The optimal time for AFP screening is 16 to 18 weeks. The triple marker screens for AFP, hCG, and unconjugated estriol. This screens for neural defects and Down syndrome. The Doppler flow study evaluates the blood flow, and amniocentesis evaluates the contents of the amniotic fluid looking for chromosomal defects.

A pregnant client is scheduled to undergo percutaneous umbilical blood sampling. The nurse understands that this test will be done in conjunction with which procedure?

amniocentesis Explanation: With percutaneous umbilical blood sampling, a sample of fetal umbilical blood is obtained through the maternal abdomen using amniocentesis technique.

A woman's obstetrician prescribes vitamin K supplements for a client who is on antiepileptic medications beginning at 36 weeks' gestation. The mother asks the nurse why she is taking this medication. The nurse's best response would be:

antiepileptic therapy can lead to vitamin K-deficient hemorrhage of the newborn. Explanation: Antiepileptic therapy may cause vitamin K-deficient hemorrhage of the newborn that the vitamin K injection the newborn receives following birth cannot fully correct. Therefore, some physicians recommend a Vitamin K supplement for their pregnant patients beginning at 36 weeks' gestation. If the mother should go into preterm labor, the newborn will have received the vitamin K prior to delivery. However, many physicians now question the usefulness of the prophylaxis.

During the newborn's assessment, which finding would lead the nurse to suspect that a large-for-gestational-age newborn has experienced birth trauma?

asymmetrical movement Explanation: A birth injury is typically characterized by asymmetrical movement. Temperature instability, seizures, and feeble sucking suggest hypoglycemia.

Encouraging routine prenatal visits is an important function for nurses to ensure the clients avoid complications or difficulties throughout the pregnancy and birth. The nurse would prepare to screen clients for gestational diabetes at which time during the pregnancy?

between 24 and 28 weeks' gestation Explanation: Screening for gestational diabetes is best done between 24 and 28 weeks' gestation, unless screening is warranted in the first trimester for high-risk reasons. If the initial screening is elevated, then further testing should be conducted to confirm the diagnosis.

A nurse working with a woman in preterm labor receives a telephone report for the fetal fibronectin test done 10 hours ago. The report indicates an absence of the protein, which the nurse knows indicates:

birth is unlikely within the 2 next weeks. Explanation: Fetal fibronectin is a protein that helps the placenta and fetal membranes adhere to the uterus during pregnancy. A negative result (absence of fetal fibronectin) is a reliable indicator that birth is unlikely within 2 weeks following the test. It does not diagnose infection.

delayed postpartum hemorrhage?

blood loss in excess of 500 ml, occurring at least 24 hours and up to 12 weeks after birth Explanation: Postpartum hemorrhage involves blood loss in excess of 500 ml. Most delayed postpartum hemorrhages occur between the fourth and ninth days postpartum, but can occur up to 12 weeks after birth The most common causes of a delayed postpartum hemorrhage include retained placental fragments, intrauterine infection, and fibroids.

The nurse is assessing a preterm newborn's fluid and hydration status. Which finding would alert the nurse to possible overhydration?

bulging fontanels Explanation: Bulging fontanels in a preterm newborn suggest overhydration. Sunken fontanels, decreased urine output, and elevated temperature would suggest dehydration.

After learning that her fetus is in the LST position, the client asks the nurse what this means. Which structure will the nurse point out is presenting first?

buttocks Explanation: The second letter indicates the presenting part. The letter "S" indicates the sacrum or buttocks as the presenting part. The fetal head would be noted by the letter "O," indicating occiput. The fetal chin would be noted by the letter "M," indicating mentum. The fetal shoulder would be noted by the letter "A," indicating the acromion process. The first letter indicates the side of the maternal pelvis toward which the presenting part is facing ("R" for right and "L" for left). The last letter specifies whether the presenting part is facing the anterior (A) or posterior (P) portion of the maternal pelvis or whether it is in a transverse (T) position.

A client with severe preeclampsia is receiving magnesium sulfate as part of the treatment plan. To ensure the client's safety, which compound would the nurse have readily available?

calcium gluconate Explanation: The woman is at risk for magnesium toxicity. The antidote for magnesium sulfate is calcium gluconate, and this should be readily available in case the woman has signs and symptoms of magnesium toxicity.

A nurse is presenting a program for a local women's group about STIs. When describing the information, the nurse would identify which infection as the most common cause of vaginal discharge?

candidiasis Explanation: Although vaginal discharge can occur with any STI, genital/vulvovaginal candidiasis is one of the most common causes of vaginal discharge. It is also referred to as yeast, monilial, and a fungal infection. It is not considered an STI because candida is a normal constituent in the vagina and becomes pathogenic only when the vaginal environment becomes altered.

The nurse is preparing a presentation for a local community group about sexually transmitted infections (STIs). Which most common STI in the United States would the nurse expect to include?

chlamydia Explanation: Chlamydia is the most common and fastest-spreading bacterial STI in the United States, with 2.8 million new cases occurring each year. Gonorrhea is the second most frequently reported communicable disease in the U.S. The incidence of syphilis had been increasing for the past 6 years. One in five people older than age 12 is infected with the virus that causes genital herpes.

During a routine assessment the nurse notes the postpartum client is tachycardic. What is a possible cause of tachycardia?

delayed hemorrhage Tachycardia in the postpartum woman can suggest anxiety, excitement, fatigue, pain, excessive blood loss or delayed hemorrhage, infection, or underlying cardiac problems. Further investigation is always warranted to rule out complications. An inability to void would suggest bladder distention. Extreme diaphoresis would be expected as the body rids itself of excess fluid. Uterine atony would be associated with a boggy uterus and excess lochia flow.

The nurse is assessing a pregnant client with a known history of congestive heart failure who is in her third trimester. Which assessment findings should the nurse prioritize?

dyspnea, crackles, and irregular weak pulse Explanation: The nurse should be alert for signs of cardiac decompensation due to congestive heart failure, which include crackles in the lungs from fluid, difficulty breathing, and weak pulse from heart exhaustion. The heart rate would not be regular, and a cough would not be dry. The heart rate would increase rather than decrease.

A multipara client develops thrombophlebitis after birth. Which assessment findings would lead the nurse to intervene immediately?

dyspnea, diaphoresis, hypotension, and chest pain Explanation: Sudden unexplained shortness of breath and reports of chest pain along with diaphoresis and hypotension suggest pulmonary embolism, which requires immediate action. Other signs and symptoms include tachycardia, apprehension, hemoptysis, syncope, and sudden change in the woman's mental status secondary to hypoxemia. Anorexia, seizures, and jaundice are unrelated to a pulmonary embolism.

A nurse in the newborn nursery has noticed that an infant is frothing and appears to have excessive drooling. Further assessment reveals that the baby has episodes of respiratory distress with choking and cyanosis. What disorder should the nurse suspect based on these findings?

esophageal atresia Explanation: Any swallowed mucus or fluid enters the blind pouch of the esophagus when a newborn suffers from esophageal atresia. The newborn with this disorder will have frothing, excessive drooling, and periods of respiratory distress with choking and cyanosis. If this happens, no feedings should be given until the newborn has been examined.

A woman is in the fourth stage of labor. During the first hour of this stage, the nurse would assess the woman's fundus at which frequency?

every 15 minutes Explanation: During the first hour of the fourth stage of labor, the nurse would assess the woman's fundus every 15 minutes and then every 30 minutes for the next hour.

The nurse is caring for a client who is considered low-risk and in active labor. During the second stage, the nurse would evaluate the client's FHR by Doppler at which frequency?

every 15 minutes Explanation: It is recommended that the FHR be assessed during the second stage of labor every 5-15 minutes min by Doppler or continuously by EFM.

A pregnant woman is diagnosed with abruptio placentae. When reviewing the woman's physical assessment in her medical record, which finding would the nurse expect?

firm, rigid uterus on palpation Explanation: The uterus is firm-to-rigid to the touch with abruptio placentae. It is soft and relaxed with placenta previa. Bleeding associated with abruptio placentae occurs suddenly and is usually dark in color. Bleeding also may not be visible. Bright red vaginal bleeding is associated with placenta previa. Fetal distress or absent fetal heart rate may be noted with abruptio placentae. The woman with abruptio placentae usually experiences constant uterine tenderness on palpation.

A pregnant client has been admitted with reports of brownish vaginal bleeding. On examination, there is an elevated human chorionic gonadotropin (hCG) level, absent fetal heart sounds, and a discrepancy between the uterine size and the gestational age. The nurse interprets these findings to suggest which condition?

gestational trophoblastic disease Explanation: The client is most likely experiencing gestational trophoblastic disease, or a molar pregnancy. In gestational trophoblastic disease, there is an abnormal proliferation and eventual degeneration of the trophoblastic villi. The signs and symptoms of molar pregnancy include brownish vaginal bleeding, elevated hCG levels, discrepancy between the uterine size and the gestational age, and absent fetal heart sounds. Placental abruption is characterized by premature separation of the placenta. Ectopic pregnancy is a condition where there is implantation of the blastocyst outside the uterus. In placenta previa, the placental attachment is at the lower uterine segment.

Assessment of a postpartum client reveals a firm uterus with bright-red bleeding and a localized bluish bulging area just under the skin at the perineum. The woman also reports significant pelvic pain and is experiencing problems with voiding. The nurse suspects which condition?

hematoma Explanation: The woman most likely has a hematoma based on the findings: firm uterus with bright-red bleeding; localized bluish bulging area just under the skin surface in the perineal area; severe perineal or pelvic pain; and difficulty voiding. A laceration would involve a firm uterus with a steady stream or trickle of unclotted bright-red blood in the perineum. Bladder distention would be palpable along with a soft, boggy uterus that deviates from the midline. Uterine atony would be noted by a uncontracted uterus.

A nurse is providing care to a client who has been diagnosed with a common benign form of gestational trophoblastic disease. The nurse identifies this as:

hydatidiform mole. Explanation: Gestational trophoblastic disease comprises a spectrum of neoplastic disorders that originate in the placenta. The two most common types are hydatidiform mole (partial or complete) and choriocarcinoma. Hydatidiform mole is a benign neoplasm of the chorion in which the chorionic villi degenerate and become transparent vesicles containing clear, viscid fluid. Ectopic pregnancy, placenta accreta, and hydramnios fall into different categories of potential pregnancy complications.

A nursing student is learning about newborn congenital defects. The defect with symptoms that include a shiny scalp, dilated scalp veins, a bulging anterior fontanel (fontanelle), and eyes pushed downward with the sclera visible above the irises is which defect?

hydrocephalus Explanation: Hydrocephalus has the clinical manifestations of a larger head than normal with widening cranial sutures. As the head enlarges, the suture lines separate, and the spaces are felt through the scalp. The anterior fontanel (fontanelle) becomes tense and bulging, the skull enlarges, the scalp becomes shiny, and its veins dilate. If pressure continues, the eyes appear to be pushed downward slightly and the sclera is visible above the irises. Spina bifida is a defect in the neural arch and is a failure of the posterior laminae of the vertebrae to close. Both septal defect and coarctation are both defects that involve the heart.

A client has been in labor for 10 hours, with contractions occurring consistently about 5 minutes apart. The resting tone of the uterus remains at about 9 mm Hg, and the strength of the contractions averages 21 mm Hg. The nurse recognizes which condition in this client?

hypotonic contractions Explanation: With hypotonic uterine contractions, the number of contractions is unusually infrequent (not more than two or three occurring in a 10-minute period). The resting tone of the uterus remains less than 10 mm Hg, and the strength of contractions does not rise above 25 mm Hg. Hypertonic uterine contractions are marked by an increase in resting tone to more than 15 mm Hg. However, the intensity of the contraction may be no stronger than that associated with hypotonic contractions. In contrast to hypotonic contractions, these occur frequently and are most commonly seen in the latent phase of labor. Uncoordinated contractions can occur so closely together they can interfere with the blood supply to the placenta. Because they occur so erratically, such as one on top of another and then a long period without any, it may be difficult for a woman to rest between contractions or to breath effectively with contractions. Braxton Hicks contractions are sporadic contractions that occur in pregnancy before the onset of true labor.

A pregnant woman diagnosed with cardiac disease 4 years ago is told that her pregnancy is a high-risk pregnancy. The nurse then explains that the danger occurs primarily because of the increase in circulatory volume. The nurse informs the client that the most dangerous time for her is when?

in weeks 28 to 32 Explanation: The danger of pregnancy in a woman with cardiac disease occurs primarily because of the increase in circulatory volume. The most dangerous time for a woman is in weeks 28 to 32, just after the blood volume peaks.

During a routine antepartal visit, a pregnant woman says, "I've noticed my gums bleeding a bit since I've become pregnant. Is this normal?" The nurse bases the response on the understanding of which effect of pregnancy?

influence of estrogen and blood vessel proliferation Explanation: During pregnancy, the gums become hyperemic, swollen, and friable and tend to bleed easily. This change is influenced by estrogen and increased proliferation of blood vessels and circulation to the mouth. Elevated progesterone levels cause smooth muscle relaxation, which results in delayed gastric emptying and decreased peristalsis. Increased venous pressure contributes to the formation of hemorrhoids. Relaxation of the cardiac sphincter, in conjunction with slowed gastric emptying, leads to reflux due to regurgitation of the stomach contents into the upper esophagus.

A client in labor has been admitted to the labor and birth suite. The nurse assessing the woman notes that the fetus is in a cephalic presentation. Which description should the nurse identify by the term presentation?

part of the fetal body entering the maternal pelvis first Explanation: The term presentation is the part of the fetal body that is entering the maternal pelvis first. The relationship of the presenting part to the sides of the maternal pelvis is called the position. Attitude is the term that describes the relation of the different fetal body parts to one another. The relation of the fetal presenting part to the maternal ischial spine is termed the station.

A couple comes in for genetic testing. They are planning to start a family and are concerned because the wife's sister has a genetic disorder. The clinic's consent form is in English and only the wife is literate in English. The nurse does not speak the client's first language. What should the nurse do?

inform the geneticist that the couple cannot give informed consent Nurses assess the client's capacity and ability to give voluntary consent. This includes assessment of factors that may interfere with informed consent, such as hearing loss, language differences, cognitive impairment, and the effects of medication. The nurse's best action is to inform the geneticist that the couple cannot give informed consent until a translator is available. The other answers are incorrect because just having the couple sign the form or explaining it in English and then having them sign the form does not allow the nurse to know that the husband understands what he is signing. The wife cannot translate for her husband because the nurse does not know if she is translating the document correctly.

After spontaneous rupture of membranes, the nurse notices a prolapsed cord. The nurse immediately places the woman in which position?

knee-chest Explanation: Pressure on the cord needs to be relieved. Therefore, the nurse would position the woman in a modified Sims, Trendelenburg, or knee-chest position. Supine, side-lying, or sitting would not provide relief of cord compression.

A nurse is assessing a preterm newborn. The nurse determines that the newborn is comfortable and without pain based on which finding?

lack of body posturing Explanation: The newborn may be in pain if the following are exhibited: sudden high-pitched cry; facial grimace with furrowing of brown and quivering chin; increased muscle tone; oxygen desaturation; body posturing, such as squirming, kicking, and arching; limb withdrawal and thrashing movements; increase in heart rate, blood pressure, pulse, and respirations; fussiness and irritability.

A client at 33 weeks' gestation is calling the office with various reports and is very concerned. The nurse recognizes which report(s) as indicating the client is potentially going into preterm labor?

low, dull backache general sense of discomfort GI upset (nausea, vomiting, diarrhea) achiness in the thighs Explanation: There are various subtle symptoms of preterm labor that a woman may experience. They include change or increase in vaginal discharge; pelvic pressure; low, dull backache; menstrual-like cramps; feeling of pelvic pressure or fullness; GI upset; general sense of discomfort or unease; heaviness or aching in the thighs; uterine contractions, with or without pain; more than six contractions per hour; intestinal cramping, with or without diarrhea; and persistent contractions.

Which medication will the nurse anticipate the health care provider will prescribe as treatment for an unruptured ectopic pregnancy?

methotrexate Explanation: Methotrexate, a folic acid antagonist that inhibits cell division in the developing embryo, is most commonly used to treat ectopic pregnancy. Oxytocin is used to stimulate uterine contractions and would be inappropriate for use with an ectopic pregnancy. Promethazine and ondansetron are antiemetics that may be used to treat hyperemesis gravidarum.

A pregnant client has a history of asthma. After reviewing the possible medications that may be prescribed during her pregnancy to control her asthma, the nurse determines additional teaching is needed when the client identifies which drug as being used?

misoprostol Explanation: Pharmacologic agents used to treat asthma in pregnancy fall into two categories: rescue agents and maintenance agents. Rescue agents provide immediate symptomatic relief by reducing acute bronchospasm. Agents used in this category include albuterol and ipratropium. Maintenance agents, by contrast, reduce the inflammation that leads to bronchospasm. Agents used in this category are inhaled steroids. Common ones prescribed include beclomethasone and salmeterol. Misoprostol is a prostaglandin that is used for treating postpartum hemorrhage but is contraindicated with asthma clients due to the risk of bronchial spasm and bronchoconstriction.

A nurse is inspecting the perineal pad of a client who gave birth vaginally to a healthy newborn 6 hours ago. The nurse observes a 5-inch stain of lochia on the pad. The nurse would document this as:

moderate. Typically, the amount of lochia is described as follows: scant-a 1- to 2-inch lochia stain on the perineal pad or approximately a 10-mL loss; light or small- an approximately 4-inch stain or a 10- to 25-mL loss; moderate- a 4- to 6-inch stain with an estimated loss of 25 to 50 mL; and large or heavy-a pad is saturated within 1 hour after changing it.

A nurse has been invited to speak at a local high school about adolescent pregnancy. When developing the presentation, the nurse would incorporate information related to which aspects?

peer pressure to become sexually active loss of self-esteem as a major impact Adolescent pregnancy has emerged as one of the most significant social problems facing our society. Early pregnancies among adolescents have major health consequences for mothers and their infants. The latest estimates show that approximately 1 million teenagers become pregnant each year in the United States, accounting for 13% of all U.S. births, but the rates have been declining in the last several years. Teen birth rates in the United States have declined but remain high, especially among African American and Hispanic teenagers and adolescents in southern states. The most important impact lies in the psychosocial area as it contributes to a loss of self-esteem, a destruction of life projects, and the maintenance of the circle of poverty. Moreover, about half of all teen pregnancies occur within 6 months of first having sexual intercourse. About one in four teen mothers under age 18 have a second baby within 2 years after the birth of the first baby.

When assessing fetal heart rate patterns, which finding would alert the nurse to a possible problem?

prolonged decelerations Explanation: Prolonged decelerations are associated with prolonged cord compression, placental abruption (abruptio placentae), cord prolapse, supine maternal position, maternal seizures, regional anesthesia, or uterine rupture. Variable decelerations are the most common deceleration pattern found. They are usually transient and correctable. Early decelerations are thought to be the result of fetal head compression. They are not indicative of fetal distress and do not require intervention. Fetal accelerations are transitory increases in FHR and provide evidence of fetal well-being.

The client at 18 weeks' gestation states, "I feel a fluttering sensation, kind of like gas." The nurse understands that the client is describing what occurrence?

quickening The fluttering sensation that can be confused with gas is called "quickening." In the 2 weeks leading up to the 20-week mark, she may feel "flutters" that she may confuse with gas. Lightening is the descent of the presenting part of the fetus into the pelvis. Placenta previa is the implantation of the placenta so that it covers part or all of the cervical os. Linea nigra is a hyperpigmented line that appears on the maternal abdomen between the symphysis pubis and top of the fundus.

The nurse is assessing the read-out of the external fetal monitor and notes late decelerations. Which action should the nurse prioritize at this time?

reposition the client on either side Explanation: Deceleration may be related to compression on the maternal abdominal aorta and inferior vena cava and repositioning the woman to either her right or left side will remove the pressure and allow the blood flow to resume. If this is not effective then the nurse would look for other potential causes such as an infusion of oxytocics. If this is unsuccessful the RN and health care provider needs to be notified immediately. The fetus is not getting enough oxygen and needs intervention. Palpating for bladder fullness would not be appropriate at this time. This is a serious situation developing and needs prompt intervention.

A multipara client in labor is having contractions which are 2 minutes apart but rarely over 50 mm Hg in strength; the resting uterine tone is high, 20 to 25 mm Hg. The client asks what she can do to make contractions more effective. The nurse's best response is:

rest between contractions. Explanation: The client's contractions are hypertonic because of the high resting tone. Hypertonic contractions occur because the uterus is being overstimulated or erratically stimulated. Rest, not activity, is effective in helping contractions become more productive. Oxytocin is contraindicated. There is no guarantee that waiting will change the status of the contractions.

A pregnant client is admitted to a maternity clinic after experiencing contractions. The assigned nurse observes that the client experiences pauses between contractions. The nurse knows that which event marks the importance of the pauses between contractions during labor?

restoration of blood flow to uterus and placenta Explanation: The pauses between contractions during labor are important because they allow the restoration of blood flow to the uterus and the placenta. Shortening of the upper uterine segment, reduction in length of the cervical canal, and effacement and dilation of the cervix are other processes that occur during uterine contractions.

A woman has been assessing her basal body temperature for 4 months. Upon reviewing her temperature history log, the nurse notes no change in her daily temperatures. Which should the nurse expect the health care provider to prescribe first?

serum progesterone level here should be a significant increase in temperature, usually 0.5° to 1° F, within a day or two after ovulation has occurred. The temperature remains elevated for 12 to 16 days, until menstruation begins. The cause of this rise in temperature is the hormone progesterone. If there is no change in the woman's monthly temperature, the progesterone level should be assessed.

A postpartum client comes to the clinic for her 6-week postpartum check up. When assessing the client's cervix, the nurse would expect the external cervical os to appear:

slit-like. After birth, the external cervical os is no longer shaped like a circle but instead appears as a jagged slit-like opening, often described as a "fish mouth."

A nurse is conducting an in-service program for a group of nurses about cervical cancer. The nurse determines that the teaching was successful when the group identifies which area as mostcommonly involved?

squamous-columnar junction Explanation: Cervical cancer starts with abnormal changes in the cellular lining or surface of the cervix. Typically these changes occur in the squamous-columnar junction of the cervix. Here, cylindrical secretory epithelial cells (columnar) meet the protective flat epithelial cells (squamous) from the outer cervix and vagina in what is termed the transformation zone.

A nurse is assessing a newborn. Which finding would alert the nurse to the possibility of respiratory distress in a newborn?

sternal retractions Explanation: Sternal retractions, cyanosis, tachypnea, expiratory grunting, and nasal flaring are signs of respiratory distress in a newborn. Symmetrical chest movements and a respiratory rate between 30 to 60 breaths/minute are typical newborn findings. Some newborns may demonstrate periodic breathing (cessation of breathing lasting 5 to 10 seconds without changes in color or heart rate) in the first few days of life.

A nurse working with victims of intimate partner violence recognizes that one of the bestinterventions is to:

strengthen the victim's sense of control over life. Explanation: The goal of intervention is to enable the victim to gain control of her life. Providing reassurance and support to the victim of intimate partner violence is key. Appropriate actions can help victims to express their thoughts and feelings in constructive ways, manage stress, and move on with their lives. These victims need to understand that they are in control, and strengthening this understanding will greatly improve their chances of making changes.

The client, who has just been walking around her room, sits down and reports leg tightness and achiness. After resting, she states she is feeling much better. The nurse recognizes that this discomfort could be due to which cause?

thromboembolic disorder of the lower extremities Explanation: Thromboembolic disorders may present with subtle changes that must be evaluated with more than just physical examination. The woman may report lower extremity tightness or aching when ambulating that is relieved with rest and elevation. Edema in the affected leg, along with warmth and tenderness and a low grade fever, may also be noted. The woman's complaints do not reflect a normal hormonal response, infection, or the body converting back to the prepregnancy state.

Review of a primiparous woman's labor and birth record reveals a prolonged second stage of labor and extended time in the stirrups. Based on an interpretation of these findings, the nurse would be especially alert for which condition?

thrombophlebitis Explanation: The woman is at risk for thrombophlebitis due to the prolonged second stage of labor, necessitating an increased amount of time in bed, and venous pooling that occurs when the woman's legs are in stirrups for a long period of time. These findings are unrelated to retained placental fragments, which would lead to uterine subinvolution, or hypertension.

A client in the third stage of labor has experienced placental separation and expulsion. Why is it necessary for a nurse to massage the woman's uterus briefly until it is firm?

to constrict the uterine blood vessels Explanation: The nurse must massage the client's uterus briefly after placental expulsion to constrict the uterine blood vessels and minimize the possibility of hemorrhage. Massaging the client's uterus will not lessen the chances of conducting an episiotomy. In addition, an episiotomy, if required, is conducted in the second stage of labor not the third. The client's uterus may appear boggy only in the fourth stage of labor not in the third stage. Ensuring that all sections of the placenta are present and that no piece is left attached to the uterine wall is confirmed through a placental examination after expulsion.

A woman with a history of crack cocaine use disorder is admitted to the labor and birth area. While caring for the client, the nurse notes a sudden onset of fetal bradycardia. Inspection of the abdomen reveals an irregular wall contour. The client also reports acute abdominal pain that is continuous. Which condition would the nurse suspect?

uterine rupture Explanation: Uterine rupture is associated with crack cocaine use disorder. Generally, the first and most reliable sign is sudden fetal distress accompanied by acute abdominal pain, vaginal bleeding, hematuria, irregular wall contour, and loss of station in the fetal presenting part. Amniotic fluid embolism often is manifested with a sudden onset of respiratory distress. Shoulder dystocia is noted when continued fetal descent is obstructed after the fetal head is delivered. Umbilical cord prolapse is noted as the protrusion of the cord alongside or ahead of the presenting part of the fetus.

The nurse is making a home visit to a woman who is 4 days postpartum. Which finding would indicate to the nurse that the woman is experiencing a problem?

uterus 1 cm below umbilicus By the fourth postpartum day, the uterus should be approximately 4 cm below the umbilicus. Being only at 1 cm indicates that the uterus is not contracting as it should. Lochia serosa is normal from days 3 to 10 postpartum. After birth the vagina is edematous and thin with few rugae. It eventually thickens and rugae return in approximately 3 weeks. Diaphoresis is common during the early postpartum period, especially in the first week. It is a mechanism to reduce fluids retained during pregnancy and restore prepregnant body fluid levels.

A nurse is caring for a client who has been prescribed gonadotropin-releasing hormone (GnRH) medication for uterine fibroids. For which side effect of GnRH medications should the nurse monitor the client?

vaginal dryness Explanation: Vaginal dryness is one of the side effects of GnRH medications. The other side effects of GnRH medications are hot flashes, headaches, mood changes, musculoskeletal malaise, bone loss, and depression. Increased vaginal discharge, urinary tract infections, and vaginitis are side effects of a pessary, not GnRH medications.

The nurse determines a newborn is small-for-gestational age based on which characteristics?

wasted appearance of extremities, thin umbilical cord, and reduced subcutaneous fat stores Explanation: The nurse should perform a thorough physical examination of the newborn and closely observe the newborn for typical SGA characteristics, which include the following: a newborn head that is disproportionately large compared with rest of body; a wasted appearance of extremities with reduced subcutaneous fat stores; a reduced amount of breast tissue; poor muscle tone over buttocks and cheeks; and a thin umbilical cord.


Related study sets